Fundamental HESI, Hesi Fundamentals, Hesi Fundamentals Practice Test, UNIT 1: Foundations of Nursing Practice

Lakukan tugas rumah & ujian kamu dengan baik sekarang menggunakan Quizwiz!

What is unintentional torts?

Negligence & Malpractice

Is there one specific route to become an RN?

No! There are various educational routes, however BSN programs are becoming the way of the profession and will soon surpass other routes.

What is the ICN definition of nursing?

Nursing encompasses autonomous and collaborative care of individuals of all ages, families, groups, & communities, sick or well in all settings.

Deontologic =

An action is right or wrong independent of its consequences.

Social justice -

Upholding moral, legal, and humanistic rights

Which nutritional assessment data should the nurse collect to best reflect total muscle mass in an adolescent? A. Height in inches or centimeters. B. Weight in kilograms or pounds. C. Triceps skin fold thickness. D. Upper arm circumference.

Upper arm circumference (D) is an indirect measure of muscle mass. (A and B) do not distinguish between fat (adipose) and muscularity. (C) is a measure of body fat. Correct Answer: D

Which intervention is most important for the nurse to implement for a male client who is experiencing urinary retention? A. Apply a condom catheter. B. Apply a skin protectant. C. Encourage increased fluid intake. D. Assess for bladder distention.

Urinary retention is the inability to void all urine collected in the bladder, which leads to uncomfortable bladder distention (D). (A and B) are useful actions to protect the skin of a client with urinary incontinence. (C) may worsen the bladder distention. Correct Answer: D

What are the steps of the nursing process?

ASSESS DIAGNOSE PLAN IMPLEMENT EVALUATE -DOCUMENT-

What are Nurse Practice Acts?

They are laws established in each state in the United States to regulate the practice of nursing.

Nursing . . .

predates written history.

Nursing is a profession dedicated to . . .

serving others.

Values =

shape our choices, behavior, and identity

The nursing process is:

systemic, dynamic, interpersonal, outcome oriented, and universally applicable.

The ICN also says that nursing care includes:

the promotion of health, prevention of illness, & the care of ill, disabled, & dying.

What are the 6 QSEN competencies?

1. Patient-centered care 2. Teamwork and collaboration 3. Quality improvement 4. Safety 5. Evidence-based practice 6. Informatics

The healthcare provider prescribes the diuretic metolazone (Zaroxolyn) 7.5 mg PO. Zaroxolyn is available in 5 mg tablets. How much should the nurse plan to administer? A. ½ tablet. B. 1 tablet. C. 1½ tablets. D. 2 tablets.

(C) is the correct calculation: D/H × Q = 7.5/5 × 1 tablet = 1½ tablets. Correct Answer: C

Blended competencies =

the set of intellectual, interpersonal, technical, and ethical/legal capacities needed to practice professional nursing

Heparin 20,000 units in 500 ml D5W at 50 ml/hour has been infusing for 5½ hours. How much heparin has the client received? A. 11,000 units. B. 13,000 units. C. 15,000 units. D. 17,000 units.

(A) is the correct calculation: 20,000 units/500 ml = 40 units (the amount of units in one ml of fluid). 40 units/ml x 50 ml/hr = 2,000 units/hour (1,000 units in 1/2 hour). 5.5 x 2,000 = 11,000 (A). OR, multiply 5 x 2,000 and add the 1/2 hour amount of 1,000 to reach the same conclusion = 11,000 units. Correct Answer: A

The healthcare provider prescribes furosemide (Lasix) 15 mg IV stat. On hand is Lasix 20 mg/2 ml. How many milliliters should the nurse administer? A. 1 ml. B. 1.5 ml. C. 1.75 ml. D. 2 ml.

(B) is the correct calculation: Dosage on hand/amount on hand = Dosage desired/x amount. 20 mg : 2 ml = 15 mg : x . 20x = 30. x = 30/20; = 1½ or 1.5 ml. Correct Answer: B

A client is to receive 10 mEq of KCl diluted in 250 ml of normal saline over 4 hours. At what rate should the nurse set the client's intravenous infusion pump? A. 13 ml/hour. B. 63 ml/hour. C. 80 ml/hour. D. 125 ml/hour.

(B) is the correct calculation: To calculate this problem correctly, remember that the dose of KCl is not used in the calculation. 250 ml/4 hours = 63 ml/hour. Correct Answer: B

At the time of the first dressing change, the client refuses to look at her mastectomy incision. The nurse tells the client that the incision is healing well, but the client refuses to talk about it. What would be an appropriate response to this client's silence? A. It is normal to feel angry and depressed, but the sooner you deal with this surgery, the better you will feel. B. Looking at your incision can be frightening, but facing this fear is a necessary part of your recovery. C. It is OK if you don't want to talk about your surgery. I will be available when you are ready. D. I will ask a woman who has had a mastectomy to come by and share her experiences with you.

(C) displays sensitivity and understanding without judging the client. (A) is judgmental in that it is telling the client how she feels and is also insensitive. (B) would give the client a chance to talk, but is also demanding and demeaning. (D) displays a positive action, but, because the nurse's personal support is not offered, this response could be interpreted as dismissing the client and avoiding the problem. Correct Answer: C

Twenty minutes after beginning a heat application, the client states that the heating pad no longer feels warm enough. What is the best response by the nurse? A. That means you have derived the maximum benefit, and the heat can be removed. B. Your blood vessels are becoming dilated and removing the heat from the site. C. We will increase the temperature 5 degrees when the pad no longer feels warm. D. The body's receptors adapt over time as they are exposed to heat.

(D) describes thermal adaptation, which occurs 20 to 30 minutes after heat application. (A and B) provide false information. (C) is not based on a knowledge of physiology and is an unsafe action that may harm the client. Correct Answer: D

An IV infusion terbutaline sulfate 5 mg in 500 ml of D5W, is infusing at a rate of 30 mcg/min prescribed for a client in premature labor. How many ml/hr should the nurse set the infusion pump? A. 30 B. 60 C. 120 D. 180

(D) is correct calculation: 180 ml/hr = 500 ml/5 mg × 1mg/1000 mcg × 30 mcg/min × 60 min/hr. Correct Answer: D

The nurse mixes 50 mg of Nipride in 250 ml of D5W and plans to administer the solution at a rate of 5 mcg/kg/min to a client weighing 182 pounds. Using a drip factor of 60 gtt/ml, how many drops per minute should the client receive? A. 31 gtt/min. B. 62 gtt/min. C. 93 gtt/min. D. 124 gtt/min.

(D) is the correct calculation: Convert lbs to kg: 182/2.2 = 82.73 kg. Determine the dosage for this client: 5 mcg × 82.73 = 413.65 mcg/min. Determine how many mcg are contained in 1 ml: 250/50,000 mcg = 200 mcg per ml. The client is to receive 413.65 mcg/min, and there are 200 mcg/ml; so the client is to receive 2.07ml per minute. With a drip factor of 60 gtt/ml, then 60 × 2.07 = 124.28 gtt/min (D) OR, using dimensional analysis: gtt/min = 60 gtt/ml X 250 ml/50 mg X 1 mg/1,000 mcg X 5 mcg/kg/min X 1 kg/2.2 lbs X 182 lbs. Correct Answer: D

The UAPs working on a chronic neuro unit ask the nurse to help them determine the safest way to transfer an elderly client with left-sided weakness from the bed to the chair. What method describes the correct transfer procedure for this client? A. Place the chair at a right angle to the bed on the client's left side before moving. B. Assist the client to a standing position, then place the right hand on the armrest. C. Have the client place the left foot next to the chair and pivot to the left before sitting. D. Move the chair parallel to the right side of the bed, and stand the client on the right foot.

(D) uses the client's stronger side, the right side, for weight-bearing during the transfer, and is the safest approach to take. (A, B, and C) are unsafe methods of transfer and include the use of poor body mechanics by the caregiver. Correct Answer: D

One can practice reflective practice in many different ways . . .

Reflection *IN* action (present) Reflection *ON* action (past) Reflection *FOR* action (future)

Healthy people 2020's primary "guidelines" are:

1. Prevent disease, disability, and premature death. 2. Having high health equity, *eliminating disparities,* and improving the health of ALL groups. 3. Create a society that promotes good health for all. 4. Promotes continued high quality of life across all lifespans.

Caring in nursing is:

- A way of being, knowing, & doing with the goal of protection, enhancement, & preservation of human dignity. - Action and competencies that aim toward the good and welfare of others.

A nurse who is working on a medical-surgical unit receives a phone call requesting information about a client who has undergone surgery. The nurse observes that the client requested a do not publish (DNP) order on any information regarding condition or presence in the hospital. What is the best response by the nurse? 1 "We have no record of that client on our unit. Thank you for calling." 2 "The new privacy laws prevent me from providing any client information over the phone." 3 "The client has requested that no information be given out. You'll need to call the client directly." 4 "It is against the hospital's policy to provide you with any information regarding any of our clients."

1 The response "We have no record of that client on our unit. Thank you for calling." conforms to the request that no information be given regarding the client's condition or presence in the hospital. HIPAA laws do not prohibit the provision of information to others as long as the client consents. The response "The client has requested that no information be given out. You'll need to call the client directly." implies that the client is admitted to the facility; this violates the client's request that no information should be shared with others. Hospital policies do not prohibit the provision of information to others as long as the client consents. The response "It is against the hospital's policy to provide you with any information regarding any of our clients." also implies that the client is admitted to the facility.

A nurse administers an intravenous solution of 0.45% sodium chloride. In what category of fluids does this solution belong? 1 Isotonic 2 Isomeric 3 Hypotonic 4 Hypertonic

1 Hypotonic solutions are less concentrated (contain less than 0.85 g of sodium chloride in each 100 mL) than body fluids. Isotonic solutions are those that cause no change in the cellular volume or pressure, because their concentration is equivalent to that of body fluid. This relates to two compounds that possess the same molecular formula but that differ in their properties or in the position of atoms in the molecules (isomers). Hypertonic solutions contain more than 0.85 g of solute in each 100 mL.

An arterial blood gas report indicates the client's pH is 7.25, PCO2 is 35 mm Hg, and HCO3 is 20 mEq/L. Which disturbance should the nurse identify based on these results? 1 Metabolic acidosis 2 Metabolic alkalosis 3 Respiratory acidosis 4 Respiratory alkalosis A low pH and low bicarbonate level are consistent with metabolic acidosis. The pH indicates acidosis. The CO2 concentration is within normal limits, which is inconsistent with respiratory acidosis; it is elevated with respiratory acidosis.

1 A low pH and low bicarbonate level are consistent with metabolic acidosis. The pH indicates acidosis. The CO2 concentration is within normal limits, which is inconsistent with respiratory acidosis; it is elevated with respiratory acidosis. Ph-7.35-7.45 PCO2 - 35-45 HCO3 - 22-30

What should the nurse consider when obtaining an informed consent from a 17-year-old adolescent? 1 If the client is allowed to give consent 2 The client cannot make informed decisions about health care. 3 If the client is permitted to give voluntary consent when parents are not available 4 The client probably will be unable to choose between alternatives when asked to consent

1 A person is legally unable to sign a consent until the age of 18 years unless the client is an emancipated minor or married. The nurse must determine the legal status of the adolescent. Although the adolescent may be capable of intelligent choices, 18 is the legal age of consent unless the client is emancipated or married. Parents or guardians are legally responsible under all circumstances unless the adolescent is an emancipated minor or married. Adolescents have the capacity to choose, but not the legal right in this situation unless they are legally emancipated or married.

After gastric surgery a client has a nasogastric tube in place. What should the nurse do when caring for this client? 1 Monitor for signs of electrolyte imbalance. 2 Change the tube at least once every 48 hours. 3 Connect the nasogastric tube to high continuous suction. 4 Assess placement by injecting 10 mL of water into the tube

1 Gastric secretions, which are electrolyte rich, are lost through the nasogastric tube; the imbalances that result can be life threatening. Changing the nasogastric tube every 48 hours is unnecessary and can damage the suture line. High continuous suction can cause trauma to the suture line. Injecting 10 mL of water into the nasogastric tube to test for placement is unsafe; if respiratory intubation has occurred aspiration will result

What type of interview is most appropriate when a nurse admits a client to a clinic? 1 Directive 2 Exploratory 3 Problem solving 4 Information giving

1 The first step in the problem-solving process is data collection so that client needs can be identified. During the initial interview a direct approach obtains specific information, such as allergies, current medications, and health history. The exploratory approach is too broad because in a nondirective interview the client controls the subject matter. Problem solving and information giving are premature at the initial visit.

Health promotion efforts with the chronically ill client should include interventions related to primary prevention. What should this include? 1 Encouraging daily physical exercise 2 Performing yearly physical examinations 3 Providing hypertension screening programs 4 Teaching a person with diabetes how to prevent complications

1 Primary prevention activities are directed toward promoting healthful lifestyles and increasing the level of well-being. Performing yearly physical examinations is a secondary prevention. Emphasis is on early detection of disease, prompt intervention, and health maintenance for those experiencing health problems. Providing hypertension screening programs is a secondary prevention. Emphasis is on early detection of disease, prompt intervention, and health maintenance for those experiencing health problems. Teaching a person with diabetes how to prevent complications is a tertiary prevention. Emphasis is on rehabilitating individuals and restoring them to an optimum level of functioning.

Heparin 20,000 units in 500 ml D5W at 50 ml/hour has been infusing for 5½ hours. How much heparin has the client received? A) 11,000 units. B) 13,000 units. C) 15,000 units. D) 17,000 units

A) 11,000 units

A nurse is caring for an older adult who is taking acetaminophen (Tylenol) for the relief of chronic pain. Which substance is most important for the nurse to determine if the client is taking because it intensifies the most serious adverse effect of acetaminophen? 1 Alcohol 2 Caffeine 3 Saw palmetto 4 St. John's wort

1 Too much ingestion of alcohol can cause scarring and fibrosis of the liver. Eighty-five to 95% of acetaminophen is metabolized by the liver. Acetaminophen and alcohol are both hepatotoxic substances. Metabolites of acetaminophen along with alcohol can cause irreversible liver damage. Caffeine affects (stimulates) the cardiovascular system, not the liver. In addition, caffeine does not interact with acetaminophen. Saw palmetto is not associated with increased liver damage when taking acetaminophen. It often is taken for benign prostatic hypertrophy because of its antiinflammatory and antiproliferative properties in prostate tissue. St. John's wort is classified as an antidepressant and is not associated with increased liver damage when taking acetaminophen. However, it does decrease the effectiveness of acetaminophen.

Twenty-four hours after a cesarean birth, a client elects to sign herself and her baby out of the hospital. Staff members are unable to contact her health care provider. The client arrives at the nursery and asks that her infant be given to her to take home. What is the most appropriate nursing action? 1 Give the infant to the client and instruct her regarding the infant's care. 2 Explain to the client that she can leave, but her infant must remain in the hospital. 3 Emphasize to the client that the infant is a minor and legally must remain until prescriptions are received. 4 Tell the client that hospital policy prevents the staff from releasing the infant until ready for discharge

1 When a client signs herself and her infant out of the hospital, she is legally responsible for her infant. The infant is the responsibility of the mother and can leave with the mother when she signs them out.

A client is taking lithium sodium (Lithium). The nurse should notify the health care provider for which of the following laboratory values? 1 White blood cell (WBC) count of 15,000 mm3 2 Negative protein in the urine 3 Blood urea nitrogen (BUN) of 20 mg/dL 4 Prothrombin of 12.0 seconds

1 White cell counts can increase with this drug. The expected range of the WBC count is 5000 to 10,000 mm3 for a healthy adult. Urinalysis, BUN, and prothrombin are not necessary and these are normal values.

What is a nurse's responsibility when administering prescribed opioid analgesics? Select all that apply. 1 Count the client's respirations. 2 Document the intensity of the client's pain. 3 Withhold the medication if the client reports pruritus. 4 Verify the number of doses in the locked cabinet before administering the prescribed dose.

1,2,4 Opioid analgesics can cause respiratory depression; the nurse must monitor respirations. The intensity of pain must be documented before and after administering an analgesic to evaluate its effectiveness. Because of the potential for abuse, the nurse is legally required to verify an accurate count of doses before taking a dose from the locked source and at the change of the shift. Pruritus is a common side effect that can be managed with antihistamines. It is not an allergic response, so it does not preclude administration. The nurse should not discard an opioid in a client's room. Any waste of an opioid must be witnessed by another nurse.

The client asks the nurse to recommend foods that might be included in a diet for diverticular disease. Which foods would be appropriate to include in the teaching plan? Select all that apply. 1 Whole grains 2 Cooked fruit and vegetables 3 Nuts and seeds 4 Lean red meats 5 Milk and eggs

1,2,5 With diverticular disease the patient should avoid foods that may obstruct the diverticuli. Therefore the fiber should be digestible, such as whole grains, and cooked fruits and vegetables. Milk and eggs have no fiber content but are good sources of protein. In clients with diverticular disease, nuts and seeds are contraindicated as they may be retained and cause inflammation and infection, which is known as diverticulitis. The client should also decrease intake of fats and red meats.

A nurse is obtaining a health history from the newly admitted client who has chronic pain in the knee. What should the nurse include in the pain assessment? Select all that apply. 1 Pain history, including location, intensity, and quality of pain 2 Client's purposeful body movement in arranging the papers on the bedside table 3 Pain pattern, including precipitating and alleviating factors 4 Vital signs such as increased blood pressure and heart rate 5 The client's family statement about increases in pain with ambulation

1,3 Accurate pain assessment includes pain history with the client's identification of pain location, intensity, and quality and helps the nurse to identify what pain means to the client. The pattern of pain includes time of onset, duration, and recurrence of pain and its assessment helps the nurse anticipate and meet the needs of the client. Assessment of the precipitating factors helps the nurse prevent the pain and determine it cause. Purposeless movements such as tossing and turning or involuntary movements such as a reflexive jerking may indicate pain. Physiological responses such as elevated blood pressure and heart rate are most likely to be absent in the client with chronic pain. Pain is a subjective experience and therefore the nurse has to ask the client directly instead of accepting statement of the family members.

A client is receiving albuterol (Proventil) to relieve severe asthma. For which clinical indicators should the nurse monitor the client? Select all that apply. 1 Tremors 2 Lethargy 3 Palpitations 4 Visual disturbances 5 Decreased pulse rate

1,3 Albuterol's sympathomimetic effect causes central nervous stimulation, precipitating tremors, restlessness, and anxiety. Albuterol's sympathomimetic effect causes cardiac stimulation that may result in tachycardia and palpitations. Albuterol may cause restlessness, irritability, and tremors, not lethargy. Albuterol may cause dizziness, not visual disturbances. Albuterol will cause tachycardia, not bradycardia.

The ultimate authority of nursing practice is . . .

the state board of nursing

A nurse is taking care of a client who has severe back pain as a result of a work injury. What nursing considerations should be made when determining the client's plan of care? Select all that apply. 1 Ask the client what is the client's acceptable level of pain. 2 Eliminate all activities that precipitate the pain. 3 Administer the pain medications regularly around the clock. 4 Use a different pain scale each time to promote patient education. 5 Assess the client's pain every 15 minutes

1,3 The nurse works together with the client in order to determine the tolerable level of pain. Considering that the client has chronic, not acute pain, the goal of the pain management is to decrease pain to the tolerable level instead of eliminating pain completely. Administration of pain medications around the clock will provide the stable level of pain medication in the blood and relieve the pain. Elimination of all activities that precipitate the client's pain is not possible even though the nurse will try to minimize such activities. The same pain scale should be used for assessment of the client's pain level helps to ensure consistency and accuracy in the pain assessment. Only management of acute pain such as postoperative pain requires the pain assessment at frequent intervals.

The nurse is preparing to administer eardrops to a client that has impacted cerumen. Before administering the drops, the nurse will assess the client for which contraindications? Select all that apply. 1 Allergy to the medication 2 Itching in the ear canal 3 Drainage from the ear canal 4 Tympanic membrane rupture 5 Partial hearing loss in the affected ear

1,3,4 Contraindications to eardrops include allergy to the medication, drainage from the ear canal, and tympanic membrane rupture. Partial hearing loss may occur with impacted cerumen and is not a contraindication to the use of eardrops. Itching may occur with some ear conditions and is not a contraindication to the use of eardrops.

A nurse is preparing to administer an ophthalmic medication to a client. What techniques should the nurse use for this procedure? Select all that apply. 1 Clean the eyelid and eyelashes. 2 Place the dropper against the eyelid. 3 Apply clean gloves before beginning of procedure. 4 Instill the solution directly onto cornea. 5 Press on the nasolacrimal duct after instilling the solution.

1,3,5 Cleaning of the eyelids and eyelashes helps to prevent contamination of the other eye and lacrimal duct. Application of gloves helps to prevent direct contact of the nurse with the client's body fluids. Applying pressure to the nasolacrimal duct prevents the medication from running out of the eye. The dropper should not touch the eyelids or eyelashes in order to prevent contamination of the medication in the dropper. The medication should not be instilled directly onto the cornea because cornea has many pain fibers and is therefore very sensitive. The medication is to be instilled into the lower conjunctival sac.

A nurse is teaching an adolescent about type 1 diabetes and self-care. Which questions from the client indicate a need for additional teaching in the cognitive domain? Select all that apply. 1 "What is diabetes?" 2 "What will my friends think?" 3 "How do I give myself an injection?" 4 "Can you tell me how the glucose monitor works?" 5 "How do I get the insulin from the vial into the syringe?

1,4 Acquiring knowledge or understanding aids in developing concepts, rather than skills or attitudes, and is a basic learning task in the cognitive domain. Values and self-realization are in the affective domain. Skills acquisition is in the psychomotor domain.

The nurse manager is planning to assign an unlicensed assistive personnel (UAP) to care for clients. What care can be delegated on a medical-surgical unit to a UAP? Select all that apply. Correct 1 Performing a bed bath for a client on bed rest 2 Evaluating the effectiveness of acetaminophen and codeine (Tylenol #3) 3 Obtaining an apical pulse rate before oral digoxin (Lanoxin) is administered Correct 4 Assisting a client who has patient-controlled analgesia (PCA) to the bathroom 5 Assessing the wound integrity of a client recovering from an abdominal laparotomy

1,4 Performing a bed bath for a client on bed rest is within the scope of practice of the UAP. Assisting a client who has PCA to the bathroom does not require professional nursing judgment and is within the job description of the UAP. Evaluating human responses to medications requires the expertise of a licensed professional nurse. Obtaining an apical pulse rate requires a professional nursing judgment to determine whether or not the medication should be administered. Evaluating human responses to health care interventions requires the expertise of a licensed professional nurse.

An older client who is receiving chemotherapy for cancer has severe nausea and vomiting and becomes dehydrated. The client is admitted to the hospital for rehydration therapy. Which interventions have specific gerontologic implications the nurse must consider? Select all that apply. 1 Assessment of skin turgor 2 Documentation of vital signs 3 Assessment of intake and output 4 Administration of antiemetic drugs 5 Replacement of fluid and electrolytes

1,4,5 When skin turgor is assessed, the presence of tenting may be related to loss of subcutaneous tissue associated with aging rather than to dehydration; skin over the sternum should be used instead of skin on the arm for checking turgor. Older adults are susceptible to central nervous system side effects, such as confusion, associated with antiemetic drugs; dosages must be reduced, and responses must be evaluated closely. Because many older adults have delicate fluid balance and may have cardiac and renal disease, replacement of fluid and electrolytes may result in adverse consequences, such as hypervolemia, pulmonary edema, and electrolyte imbalance. Vital signs can be obtained as with any other adult. Intake and output can be measured accurately in older adults.

What are the professional values of a nurse?

1. Altruism 2. Autonomy 3. Human dignity 4. Integrity 5. Social justice

What is the principle-based approach to bioethics?

1. Autonomy 2. Nonmaleficence 3. Beneficence 4. Justice 5. Fidelity 6. Veracity, accountability, privacy, and confidentiality

What are the roles of a nurse?

1. Caregiver 2. Communicator 3. Educator 4. Counselor 5. Leader 6: Researcher 7. Advocate 8. Collaborator

What are the six essential features of professional nursing? (Generally speaking).

1. Caring relationships that facilitate health and healing. 2. Being aware of the range of human responses to health and illness in their various environments. 3. Integrating objective date with the patient's or groups subjective experience. 4.Applying scientific knowledge to care for the patient, through the use of critical thinking. 5. Learning through scholarly inquiry. 6. Influence on the promotion of social justice.

What are the four elements of liability?

1. Duty. 2. Breach of Duty. 3. Causation 4. Damages.

What are the FOUR aims of nursing?

1. To promote health. 2. To prevent illness. 3. To restore health (alleviate suffering). 4. To facilitate coping with disability or death.

What are the four blended competencies of nursing?

1. cognitive 2. technical 3. interpersonal 4. ethical/legal

A health care provider prescribes 500 mg of an antibiotic intravenous piggyback (IVPB) every 12 hours. The vial of antibiotic contains 1 g and indicates that the addition of 2.5 mL of sterile water will yield 3 mL of reconstituted solution. How many milliliters of the antibiotic should be added to the 50 mL IVPB bag? Record your answer using one decimal place. __ mL

1.5

Seconal 0.1 gram PRN at bedtime is prescribed to a client for rest. The scored tablets are labeled grain 1.5 per tablet. How many tablets should the nurse plan to administer? A. 0.5 tablet. B. 1 tablet. C. 1.5 tablets. D. 2 tablets.

15 gr=1 Gm. Converting the prescribed dose of 0.1 grams to grains requires multiplying 0.1 × 15 = 1.5 grains. The tablets come in 1.5 grains, so the nurse should plan to administer 1 tablet (B). Correct Answer: B

Atropine Conduction is slow, rate can be normal

1st degree av block

Which drug does a nurse anticipate may be prescribed to produce diuresis and inhibit formation of aqueous humor for a client with glaucoma? 1 Chlorothiazide (Diuril) 2 Acetazolamide (Diamox) 3 Bendroflumethiazide (Naturetin) 4 Demecarium bromide (Humorsol)

2

A client who is suspected of having tetanus asks a nurse about immunizations against tetanus. Before responding, what should the nurse consider about the benefits of tetanus antitoxin? 1 It stimulates plasma cells directly. 2 A high titer of antibodies is generated. 3 It provides immediate active immunity. 4

2 A long-lasting passive immunity is produced. Tetanus antitoxin provides antibodies, which confer immediate passive immunity. Antitoxin does not stimulate production of antibodies. It provides passive, not active, immunity. Passive immunity, by definition, is not long-lasting.

While undergoing a soapsuds enema, the client reports abdominal cramping. What action should the nurse take? 1 Immediately stop the infusion. 2 Lower the height of the enema bag. 3 Advance the enema tubing 2 to 3 inches. 4 Clamp the tube for 2 minutes, then restart the infusion.

2 Abdominal cramping during a soapsuds enema may be due to too rapid administration of the enema solution. Lowering the height of the enema bag slows the flow and allows the bowel time to adapt to the distention without causing excessive discomfort. Stopping the infusion is not necessary. Advancing the enema tubing is not appropriate. Clamping the tube for several minutes then restarting the infusion may be attempted if slowing the infusion does not relieve the cramps.

During the initial physical assessment of a newly admitted client with a pressure ulcer, a nurse observes that the client's skin is dry and scaly. The nurse applies emollients and reinforces the dressing on the pressure ulcer. Legally, were the nurse's actions adequate? 1 The nurse also should have instituted a plan to increase activity. 2 The nurse provided supportive nursing care for the well-being of the client. 3 Debridement of the pressure ulcer should have been done before the dressing was applied. 4 Treatment should not have been instituted until the health care provider's prescriptions were received.

2 According to the Nurse Practice Act, a nurse may independently treat human responses to actual or potential health problems. An activity level is prescribed by a health care provider; this is a dependent function of the nurse. There is not enough information to come to the conclusion that debridement should have been done before the dressing was applied. Application of an emollient and reinforcing a dressing are independent nursing functions.

A nurse is preparing to administer an oil-retention enema and understands that it works primarily by: 1 Stimulating the urge to defecate. 2 Lubricating the sigmoid colon and rectum. 3 Dissolving the feces. 4 Softening the feces

2 The primary purpose of an oil-retention enema is to lubricate the sigmoid colon and rectum. Secondary benefits of an oil-retention enema include stimulating the urge to defecate and softening feces. An oil-retention enema does not dissolve feces.

A client is receiving an intravenous (IV) infusion of 5% dextrose in water. The client loses weight and develops a negative nitrogen balance. The nurse concludes that what likely contributed to this client's weight loss? 1 Excessive carbohydrate intake 2 Lack of protein supplementation 3 Insufficient intake of water-soluble vitamins 4 Increased concentration of electrolytes in cells

2 An infusion of dextrose in water does not provide proteins required for tissue growth, repair, and maintenance; therefore, tissue breakdown occurs to supply the essential amino acids. Each liter provides approximately 170 calories, which is insufficient to meet minimal energy requirements; tissue breakdown will result. Weight loss is caused by insufficient nutrient intake; vitamins do not prevent weight loss. An infusion of 5% dextrose in water may decrease electrolyte concentration.

A toddler screams and cries noisily after parental visits, disturbing all the other children. When the crying is particularly loud and prolonged, the nurse puts the crib in a separate room and closes the door. The toddler is left there until the crying ceases, a matter of 30 or 45 minutes. Legally, how should this behavior be interpreted? 1 Limits had to be set to control the child's crying. 2 The child had a right to remain in the room with the other children. 3 The child had to be removed because the other children needed to be considered. 4 Segregation of the child for more than half an hour was too long a period of time

2 Legally, a person cannot be locked in a room (isolated) unless there is a threat of danger either to the self or to others. Limit setting in this situation is not warranted. This is a reaction to separation from the parent, which is common at this age. Crying, although irritating, will not harm the other children. A child should never be isolated

A nurse is caring for a client with an impaired immune system. Which blood protein associated with the immune system is important for the nurse to consider? 1 Albumin 2 Globulin 3 Thrombin 4 Hemoglobin

2 The gamma-globulin fraction in the plasma is the fraction that includes the antibodies. Albumin helps regulate fluid shifts by maintaining plasma oncotic pressure. Thrombin is involved with clotting. Hemoglobin carries oxygen.

What clinical indicators should the nurse expect a client with hyperkalemia to exhibit? Select all that apply. 1 Tetany 2 Seizures 3 Diarrhea 4 Weakness 5 Dysrhythmias

3,4,5 Tetany is caused by hypocalcemia. Seizures caused by electrolyte imbalances are associated with low calcium or sodium levels. Because of potassium's role in the sodium/potassium pump, hyperkalemia will cause diarrhea, weakness, and cardiac dysrhythmias.

In what position should the nurse place a client recovering from general anesthesia? 1 Supine Correct2 Side-lying 3 High Fowler 4 Trendelenburg

2 Turning the client to the side promotes drainage of secretions and prevents aspiration, especially when the gag reflex is not intact. This position also brings the tongue forward, preventing it from occluding the airway when it is in the relaxed state. The risk for aspiration is increased when the supine position is assumed by a semi-alert client. High Fowler position may cause the neck to flex in a client who is not alert, interfering with respirations. Trendelenburg position is not used for a postoperative client because it interferes with breathing.

A client is being admitted for a total hip replacement. When is it necessary for the nurse to ensure that a medication reconciliation is completed? Select all that apply. 1 After reporting severe pain 2 On admission to the hospital 3 Upon entering the operating room 4 Before transfer to a rehabilitation facility 5 At time of scheduling for the surgical procedure

2, 4 Medication reconciliation involves the creation of a list of all medications the client is taking and comparing it to the health care provider's prescriptions on admission or when there is a transfer to a different setting or service, or discharge. A change in status does not require medication reconciliation. A medication reconciliation should be completed long before entering the operating room. Total hip replacement is elective surgery, and scheduling takes place before admission; medication reconciliation takes place when the client is admitted.

An 85-year-old client has just been admitted to a nursing home. When designing a plan of care for this older adult the nurse recalls what expected sensory losses associated with aging? Select all that apply. 1 Difficulty in swallowing 2 Diminished sensation of pain 3 Heightened response to stimuli 4 Impaired hearing of high-frequency sounds 5 Increased ability to tolerate environmental heat

2,4 Because of aging of the nervous system an older adult has a diminished sensation of pain and may be unaware of a serious illness, thermal extremes, or excessive pressure. As people age they experience atrophy of the organ of Corti and cochlear neurons, loss of the sensory hair cells, and degeneration of the stria vascularis, which affects an older person's ability to perceive high-frequency sounds. An interference with swallowing is a motor, not a sensory, loss, nor is it an expected response to aging. There is a decreased, not heightened, response to stimuli in older adults. There is a decreased, not increased, ability to physiologically adjust to extremes in environmental temperature.

The nurse expects a client with an elevated temperature to exhibit what indicators of pyrexia? Select all that apply. Incorrect 1 Dyspnea 2 Flushed face 3 Precordial pain 4 Increased pulse rate 5 Increased blood pressure

2,4 Increased body heat dilates blood vessels, causing a flushed face. The pulse rate increases to meet increased tissue demands for oxygen in the febrile state. Fever may not cause difficult breathing. Pain is not related to fever. Blood pressure is not expected to increase with fever.

The nurse recognizes that which are important components of a neurovascular assessment? Select all that apply. 1 Orientation 2 Capillary refill 3 Pupillary response 4 Respiratory rate 5 Pulse and skin temperature 6 Movement and sensation

2,5,6, A neurovascular assessment involves evaluation of nerve and blood supply to an extremity involved in an injury. The area involved may include an orthopedic and/or soft tissue injury. A correct neurovascular assessment should include evaluation of capillary refill, pulses, warmth and paresthesias, and movement and sensation. Orientation, pupillary response, and respiratory rate are components of a neurological assessment.

To minimize the side effects of the vincristine (Oncovin) that a client is receiving, what does the nurse expect the dietary plan to include? 1 Low in fat 2 High in iron 3 High in fluids 4 Low in residue

3 A common side effect of vincristine is a paralytic ileus that results in constipation. Preventative measures include high-fiber foods and fluids that exceed minimum requirements. These will keep the stool bulky and soft, thereby promoting evacuation. Low in fat, high in iron, and low in residue dietary plans will not provide the roughage and fluids needed to minimize the constipation associated with vincristine.

A nurse is providing care to a client eight hours after the client had surgery to correct an upper urinary tract obstruction. Which assessment finding should the nurse report to the surgeon? 1 Incisional pain 2 Absent bowel sounds 3 Urine output of 20 mL/hour 4 Serosanguineous drainage on the dressing .

3 A urinary output of 50 mL/hr or greater is necessary to prevent stasis and consequent infections after this type of surgery. The nurse should notify the surgeon of the assessment findings, since this may indicate a urinary tract obstruction. Incisional pain, absent bowel sounds, and serosanguineous drainage are acceptable assessment findings for this client after this procedure and require continued monitoring but do not necessarily require reporting to the surgeon

The nurse is providing postoperative care to a client who had a submucosal resection (SMR) for a deviated septum. The nurse should monitor for what complication associated with this type of surgery? Incorrect1 Occipital headache 2 Periorbital crepitus 3 Expectoration of blood 4 Changes in vocalization

3 After an SMR, hemorrhage from the area should be suspected if the client is swallowing frequently or expelling blood with saliva. A headache in the back of the head is not a complication of a submucosal resection. Crepitus is caused by leakage of air into tissue spaces; it is not an expected complication of SMR. The nerves and structures involved with speech are not within the operative area. However, the sound of the voice is altered temporarily by the presence of nasal packing and edema.

Autonomy in bioethics =

Respect rights of patients to make health care decisions

The triage nurse in the emergency department receives four clients simultaneously. Which of the clients should the nurse determine can be treated last? 1 Multipara in active labor 2 Middle-aged woman with substernal chest pain 3 Older adult male with a partially amputated finger 4 Adolescent boy with an oxygen saturation of 91%

3 Although a client with a partially amputated finger needs control of bleeding, the injury is not life threatening and the client can wait for care. A woman in active labor should be assessed immediately because birth may be imminent. A woman with chest pain may be experiencing a life-threatening illness and should be assessed immediately. An adolescent with significant hypoxia may be experiencing a life-threatening illness and should be assessed immediately.

The nurse is caring for a client that is on a low carbohydrate diet. With this diet, there is decreased glucose available for energy, and fat is metabolized for energy resulting in an increased production of which substance in the urine? 1 Protein 2 Glucose 3 Ketones 4 Uric Acid

3 As a result of fat metabolism, ketone bodies are formed and the kidneys attempt to decrease the excess by filtration and excretion. Excessive ketones in the blood can cause metabolic acidosis. A low carbohydrate diet does not cause increased protein, glucose, or uric acid in the urine. Study Tip: The old standbys of enough sleep and adequate nutritional intake also help keep excessive stress at bay. Although nursing students learn about the body's energy needs in anatomy and physiology classes, somehow they tend to forget that glucose is necessary for brain cells to work. Skipping breakfast or lunch or surviving on junk food puts the brain at a disadvantage.

A client comes to the clinic complaining of a productive cough with copious yellow sputum, fever, and chills for the past two days. The first thing the nurse should do when caring for this client is to: 1 Encourage fluids. 2 Administer oxygen. 3 Take the temperature. 4 Collect a sputum specimen

3 Baseline vital signs are extremely important; physical assessment precedes diagnostic measures and intervention. This is done after the health care provider makes a medical diagnosis; this is not an independent function of the nurse. Encouraging fluids might be done after it is determined whether a specimen for blood gases is needed; this is not usually an independent function of the nurse. Oxygen is administered independently by the nurse only in an emergency situation. A sputum specimen should be obtained after vital signs and before administration of antibiotics.

A nurse is caring for a client diagnosed with methicillin-resistant Staphylococcus aureus (MRSA) in the urine. The health care provider orders an indwelling urinary catheter to be inserted. Which precaution should the nurse take during this procedure? 1 Droplet precautions 2 Reverse isolation 3 Surgical asepsis 4 Medical asepsis

3 Catheter insertion requires the procedure to be performed under sterile technique . Droplet precautions are used with certain respiratory illnesses. Reverse isolation is used with clients who may be immunocompromised. Medical asepsis involves clean technique/gloving.

A client has been admitted with a urinary tract infection. The nurse receives a urine culture and sensitivity report that reveals the client has vancomycin-resistant Entercoccus (VRE). After notifying the physician, which action should the nurse take to decrease the risk of transmission to others? 1 Insert a urinary catheter. 2 Initiate droplet precautions. 3 Move the client to a private room. 4 Use a high efficiency particulate air (HEPA) respirator during care.

3 Contact precautions are used for clients with known or suspected infections transmitted by direct contact or contact with items in the environment; therefore infectious clients must be placed in a private room. There is no need to insert an indwelling catheter, as this can increase the risk for additional infection. Droplet precautions are used for clients known or suspected to have infections transmitted by the droplet route. These infections are caused by organisms in droplets that may travel 3 feet, but are not suspended for long periods.

The nurse is preparing discharge instructions for a client who has begun to demonstrate signs of early Alzheimer dementia. The client lives alone. The client's adult children live nearby. According to the prescribed medication regimen the client is to take medications six times throughout the day. What is the priority nursing intervention to assist the client with taking the medication? 1 Contact the client's children and ask them to hire a private duty aide who will provide round-the-clock care. 2 Develop a chart for the client, listing the times the medication should be taken. 3 Contact the primary health care provider and discuss the possibility of simplifying the medication regimen. 4 Instruct the client and client's children to put medications in a weekly pill organizer

3 Contacting a medical care provider and discussing the possibility of simplifying the client's medication regimen will make it possible to use a weekly pill organizer : an empty pill box will remind the client who has a short-term memory deficit due to Alzheimer dementia that medication was taken and will prevent medication being taken multiple times. The client does not require 24-hour supervision because the client is in the outset of the Alzheimer dementia and the major issue is a short-term memory loss. A chart may be complex and difficult to understand for the client and will require the client to perform cognitive tasks multiple times on daily basis that may be beyond the client's ability. Use of the weekly pill organizers will be difficult with the current medication regimen when the client has to take medications six times a day; the medication regimen has to be simplified first.

A client has an anaphylactic reaction after receiving intravenous penicillin. What does the nurse conclude is the cause of this reaction? 1 An acquired atopic sensitization occurred. 2 There was passive immunity to the penicillin allergen. 3 Antibodies to penicillin developed after a previous exposure. 4 Potent antibodies were produced when the infusion was instituted

3 Hypersensitivity results from the production of antibodies in response to exposure to certain foreign substances (allergens). Earlier exposure is necessary for the development of these antibodies. This is not a sensitivity reaction to penicillin; hay fever and asthma are atopic conditions. It is an active, not passive, immune response. Antibodies developed when there was a previous, not current, exposure to penicillin.

The nurse should instruct a client with an ileal conduit to empty the collection device frequently because a full urine collection bag may: 1 Force urine to back up into the kidneys. 2 Suppress production of urine. 3 Cause the device to pull away from the skin. 4 Tear the ileal conduit

3 If the device becomes full and is not emptied, it may pull away from the skin and leak urine. Urine in contact with unprotected skin will irritate and cause skin breakdown. A full urine collection bag will not cause urine to back up into the kidneys, suppress the production of urine, or tear the ileal conduit.

A client is to have mafenide (Sulfamylon) cream applied to burned areas. For which serious side effect of mafenide therapy should the nurse monitor this client? 1 Curling ulcer 2 Renal shutdown 3 Metabolic acidosis 4 Hemolysis of red blood cells

3 Mafenide interferes with the kidneys' role in hydrogen ion excretion, resulting in metabolic acidosis. Curling ulcer, renal shutdown, and hemolysis of red blood cells are not adverse effects of the drugs.

The nurse reviews a medical record and is concerned that the client may develop hyperkalemia. Which disease increases the risk of hyperkalemia? 1 Crohn's 2 Cushing's 3 End-stage renal 4 Gastroesophageal reflux .

3 One of the kidneys' functions is to eliminate potassium from the body; diseases of the kidneys often interfere with this function, and hyperkalemia may develop, necessitating dialysis. Clients with Crohn's disease have diarrhea, resulting in potassium loss. Clients with Cushing's disease will retain sodium and excrete potassium. Clients with gastroesophageal reflux disease are prone to vomiting that may lead to sodium and chloride loss with minimal loss of potassium

When a client files a lawsuit against a nurse for malpractice, the client must prove that there is a link between the harm suffered and actions performed by the nurse that were negligent. This is known as: 1 Evidence 2 Tort discovery 3 Proximate cause 4 Common cause

3 Proximate cause is the legal concept meaning that the client must prove that the nurse's actions contributed to or caused the client's injury. Evidence is data presented in proof of the facts, which may include witness testimony, records, documents, or objects. A tort is a wrongful act, not including a breach of contract of trust that results in injury to another person. Common cause means to unite one's interest with another's.

What is a basic concept associated with rehabilitation that the nurse should consider when formulating discharge plans for clients? 1 Rehabilitation needs are met best by the client's family and community resources. Incorrect2 Rehabilitation is a specialty area with unique methods for meeting clients' needs. Correct3 Immediate or potential rehabilitation needs are exhibited by clients with health problems. 4 Clients who are returning to their usual activities following hospitalization do not require rehabilitation.

3 Rehabilitation refers to a process that assists clients to obtain optimal functioning. Care should be initiated immediately when a health problem exists to avoid complications and facilitate recuperation. All resources that can be beneficial to client rehabilitation, including the private health care provider and acute care facilities, should be used. Rehabilitation is a commonality in all areas of nursing practice. Rehabilitation is necessary to help clients return to a previous or optimal level of functioning.

A nurse discusses the philosophy of Alcoholics Anonymous (AA) with the client who has a history of alcoholism. What need must self-help groups such as AA meet to be successful? 1 Trust 2 Growth 3 Belonging 4 Independence

3 Self-help groups are successful because they support a basic human need for acceptance. A feeling of comfort and safety and a sense of belonging may be achieved in a nonjudgmental, supportive, sharing experience with others. AA meets dependency needs rather than focusing on independence, trust, and growth.

used to treat ventricular ectopy, ventricular tachycardia, and ventricular fibrillation

Lidocaine

A nurse receives a subpoena in a court case involving a child. The nurse is preparing to appear in court. In addition to the state Nurse Practice Act and the American Nursing Association (ANA) Code for Nurses, what else should the nurse review? 1 Nursing's Social Policy Statement 2 State law regarding protection of minors 3 ANA Standards of Clinical Nursing Practice 4 References regarding a child's right to consent

3 The ANA Standards of Clinical Nursing Practice guidelines govern safe nursing practice; nurses are legally responsible to perform according to these guidelines. Nursing's Social Policy Statement explains what the public can expect from nurses, but it is not used to govern nursing practice. There are no data that indicate state law regarding protection of minors and references regarding a child's right to consent are necessary.

A nurse cares for a client that has been bitten by a large dog. A bite by a large dog can cause which type of trauma? 1 Abrasion 2 Fracture 3 Crush injury 4 Incisional laceration

3 The bite of a large dog can exert between 150 and 400 psi of pressure, causing a crush injury. A crush injury may or may not include a fracture. Abrasions and incisional lacerations are not caused by this form of trauma.

A client has undergone a subtotal thyroidectomy. The client is being transferred from the post anesthesia care unit/recovery area to the inpatient nursing unit. What emergency equipment is most important for the nurse to have available for this client? 1 A defibrillator 2 An IV infusion pump 3 A tracheostomy tray 4 An electrocardiogram (ECG) monitor

3 The client who has undergone a subtotal thyroidectomy is at high risk for airway occlusion resulting from postoperative edema. With this in mind, emergency airway equipment such as a tracheostomy set and intubation supplies should be immediately available to the client. A defibrillator, an IV infusion pump, and an electrocardiogram (ECG) monitor are all equipment items that should be available to all postoperative clients.

A client asks about the purpose of a pulse oximeter. The nurse explains that it is used to measure the: 1 Respiratory rate. 2 Amount of oxygen in the blood. 3 Percentage of hemoglobin-carrying oxygen. 4 Amount of carbon dioxide in the blood

3 The pulse oximeter measures the oxygen saturation of blood by determining the percentage of hemoglobin-carrying oxygen. A pulse oximeter does not interpret the amount of oxygen or carbon dioxide carried in the blood, nor does it measure respiratory rate.

A client reaches the point of acceptance during the stages of dying. What response should the nurse expect the client to exhibit? 1 Apathy 2 Euphoria 3 Detachment 4 Emotionalism

3 When an individual reaches the point of being intellectually and psychologically able to accept death, anxiety is reduced and the individual becomes detached from the environment. Although detached, the client is not apathetic but still may be concerned and use time constructively. Although resigned to death, the individual is not euphoric. In the stage of acceptance, the client is no longer angry or depressed.

A nurse is preparing a community health program for senior citizens. The nurse teaches the group that the physical findings that are typical in older people include: 1 A loss of skin elasticity and a decrease in libido 2 Impaired fat digestion and increased salivary secretions 3 Increased blood pressure and decreased hormone production 4 An increase in body warmth and some swallowing difficulties

3 With aging, narrowing of the arteries causes some increase in the systolic and diastolic blood pressures; hormone production decreases after menopause. There may or may not be changes in libido; there is a loss of skin elasticity. Salivary secretions decrease, not increase, causing more difficulty with swallowing; there is some impairment of fat digestion. There may be a decrease in subcutaneous fat and decreasing body warmth; some swallowing difficulties occur because of decreased oral secretions.

Which age-related change should the nurse consider when formulating a plan of care for an older adult? Select all that apply. Incorrect 1 Difficulty in swallowing 2 Increased sensitivity to heat 3 Increased sensitivity to glare 4 Diminished sensation of pain 5 Heightened response to stimuli .

3,4 Changes in the ciliary muscles, decrease in pupil size, and a more rigid pupil sphincter contribute to an increased sensitivity to glare. Diminished sensation of pain may make an older individual unaware of a serious illness, thermal extremes, or excessive pressure. There should be no interference with swallowing in older individuals. Older individuals tend to feel the cold and rarely complain of the heat. There is a decreased response to stimuli in the older individual

A client is admitted with severe diarrhea that resulted in hypokalemia. The nurse should monitor for what clinical manifestations of the electrolyte deficiency? Select all that apply. 1 Diplopia 2 Skin rash 3 Leg cramps 4 Tachycardia 5 Muscle weakness

3,5 Leg cramps occur with hypokalemia because of potassium deficit. Muscle weakness occurs with hypokalemia because of the alteration in the sodium potassium pump mechanism. Diplopia does not indicate an electrolyte deficit. A skin rash does not indicate an electrolyte deficit. Tachycardia is not associated with hypokalemia, bradycardia is.

An intravenous piggyback (IVPB) of cefazolin (Kefzol) 500 mg in 50 mL of 5% dextrose in water is to be administered over a 20-minute period. The tubing has a drop factor of 15 drops/mL. At what rate per minute should the nurse regulate the infusion to run? Record the answer using a whole number. ______ gtts/min Solve the problem by using the following formula: Drops per minute = total number of drops / total time in minutes Drops per minute = 50 mL x 15 (drop factor) / 20 mintes = 750 / 20 = 37.5. Round the answer to 38 drops per minute.

38 Solve the problem by using the following formula: Drops per minute = total number of drops / total time in minutes Drops per minute = 50 mL x 15 (drop factor) / 20 mintes = 750 / 20 = 37.5. Round the answer to 38 drops per minute

A client receiving steroid therapy states, "I have difficulty controlling my temper which is so unlike me, and I don't know why this is happening." What is the nurse's best response? 1 Tell the client it is nothing to worry about. 2 Talk with the client further to identify the specific cause of the problem. 3 Instruct the client to attempt to avoid situations that cause irritation. 4 Interview the client to determine whether other mood swings are being experienced.

4

A client receiving steroid therapy states, "I have difficulty controlling my temper which is so unlike me, and I don't know why this is happening." What is the nurse's best response? 1 Tell the client it is nothing to worry about. 2 Talk with the client further to identify the specific cause of the problem. 3 Instruct the client to attempt to avoid situations that cause irritation. 4 Interview the client to determine whether other mood swings are being experienced.

4

The nurse is caring for a client with a temperature of 104.5 degrees Fahrenheit. The nurse applies a cooling blanket and administers an antipyretic medication. The nurse explains that the rationale for these interventions is to: 1 Promote equalization of osmotic pressures. 2 Prevent hypoxia associated with diaphoresis. 3 Promote integrity of intracerebral neurons. 4 Reduce brain metabolism and limit hypoxia.

4

The nurse is caring for a non-ambulatory client with a reddened sacrum that is unrelieved by repositioning. What nursing diagnosis should be included on the client's plan of care? 1 Risk for pressure ulcer 2 Risk for impaired skin integrity 3 Impaired skin integrity, related to infrequent turning and repositioning 4 Impaired skin integrity, related to the effects of pressure and shearing force

4

A health care provider prescribes 10 mL of a 10% solution of calcium gluconate for a client with a severely depressed serum calcium level. The client also is receiving digoxin (Lanoxin) 0.25 mg daily and an intravenous (IV) solution of D5W. The nurse's next action is based on the fact that calcium gluconate: 1 Can be added to any IV solution Incorrect2 Must be administered via an intravenous piggyback (IVPB) 3 Is non-irritating to surrounding tissues Correct4 Potentiates the action of the digoxin preparation

4 Toxicity can result because the action of calcium ions is similar to that of digoxin. Calcium gluconate cannot be added to a solution containing carbonate or phosphate because a dangerous precipitation will occur. Calcium gluconate can be added to the IV solution the client is receiving. If calcium infiltrates, sloughing of tissue will result.

A client has been diagnosed as brain dead. The nurse understands that this means that the client has: 1 No spontaneous reflexes 2 Shallow and slow breathing 3 No cortical functioning with some reflex breathing 4 Deep tendon reflexes only and no independent breathing

4 A client who is declared as being brain dead has no function of the cerebral cortex and a flat EEG. The client may have some spontaneous breathing and a heartbeat. The guidelines established by the American Association of Neurology include coma or unresponsiveness, absence of brainstem reflexes, and apnea. There are specific assessments to validate the findings. The other answer options do not fit the definition of brain dead.

A client has a pressure ulcer that is full thickness with necrosis into the subcutaneous tissue down to the underlying fascia. The nurse should document the assessment finding as which stage of pressure ulcer? 1 Stage I 2 Stage II 3 Stage III 4 Unstageable

4 A pressure ulcer with necrotic tissue is unstageable. The necrotic tissue must be removed before the wound can be staged. A stage I pressure ulcer is defined as an area of persistent redness with no break in skin integrity. A stage II pressure ulcer is a partial-thickness wound with skin loss involving the epidermis, dermis, or both; the ulcer is superficial and may present as an abrasion, blister, or shallow crater. A stage III pressure ulcer involves full thickness tissue loss with visible subcutaneous fat. Bone, tendon, and muscle are not exposed.

Autonomy =

Standing alone; independence; right to self-determination

What is the maximum length of time a nurse should allow an intravenous (IV) bag of solution to infuse? 1 6 hours 2 12 hours 3 18 hours 4 24 hours

4 After 24 hours there is increased risk for contamination of the solution and the bag should be changed. It is unnecessary to change the bag any less often.

A nurse is reviewing a plan of care for a client who was admitted with dehydration as a result of prolonged watery diarrhea. Which prescription should the nurse question? 1 Oral psyllium (Metamucil) 2 Oral potassium supplement 3 Parenteral half normal saline 4 Parenteral albumin (Albuminar)

4 Albumin is hypertonic and will draw additional fluid from the tissues into the intravascular space. Oral psyllium will absorb the watery diarrhea, giving more bulk to the stool. An oral potassium supplement is appropriate because diarrhea causes potassium loss. Parenteral half normal saline is a hypotonic solution, which can correct dehydration.

To ensure the safety of a client who is receiving a continuous intravenous normal saline infusion, the nurse should change the administration set every: 1 4 to 8 hours 2 12 to 24 hours 3 24 to 48 hours 4 72 to 96 hours

4 Best practice guidelines recommend replacing administration sets no more frequently than 72 to 96 hours after initiation of use in patients not receiving blood, blood products, or fat emulsions. This evidence-based practice is safe and cost effective. Changing the administration set every 4 to 48 hours is not a cost-effective practice

A dying client is coping with feelings regarding impending death. The nurse bases care on the theory of death and dying by Kübler-Ross. During which stage of grieving should the nurse primarily use nonverbal interventions? 1 Anger 2 Denial 3 Bargaining 4 Acceptance

4 Communication and interventions during the acceptance stage are mainly nonverbal (e.g., holding the client's hand). The nurse should be quiet but available. During the anger stage the nurse should accept that the client is angry. The anger stage requires verbal communication. During the denial stage the nurse should accept the client's behavior but not reinforce the denial. The denial stage requires verbal communication. During the bargaining stage the nurse should listen intently but not provide false reassurance. The bargaining stage requires verbal communication.

A health care provider has prescribed isoniazid (Laniazid) for a client. Which instruction should the nurse give the client about this medication? 1 Prolonged use can cause dark concentrated urine. 2 The medication is best absorbed when taken on an empty stomach. 3 Take the medication with aluminum hydroxide to minimize GI upset. 4 Drinking alcohol daily can cause drug-induced hepatitis

4 Daily alcohol intake can cause drug induced hepatitis. Prolonged use does not cause dark concentrated urine. The client should take isoniazid with meals to decrease GI upset. Clients should avoid taking aluminum antacids at the same time as this medication because it impairs absorption.

When being interviewed for a position as a registered professional nurse, the applicant is asked to identify an example of an intentional tort. What is the appropriate response? 1 Negligence 2 Malpractice 3 Breach of duty 4 False imprisonment

4 False imprisonment is a wrong committed by one person against another in a willful, intentional way without just cause or excuse. Negligence is an unintentional tort. Malpractice, which is professional negligence, is classified as an unintentional tort. Breach of duty is an unintentional tort.

A client who was exposed to hepatitis A asks why an injection of gamma globulin is needed. Before responding, what should the nurse consider about how gamma globulin provides passive immunity? 1 It increases production of short-lived antibodies. 2 It accelerates antigen-antibody union at the hepatic sites. 3 The lymphatic system is stimulated to produce antibodies. 4 The antigen is neutralized by the antibodies that it supplies

4 Gamma globulin, which is an immune globulin, contains most of the antibodies circulating in the blood. When injected into an individual, it prevents a specific antigen from entering a host cell. Gamma globulin does not stimulate antibody production. It does not affect antigen-antibody function.

Often when a family member is dying, the client and the family are at different stages of grieving. During which stage of a client's grieving is the family likely to require more emotional nursing care than the client? 1 Anger 2 Denial 3 Depression 4 Acceptance

4 In the stage of acceptance, the client frequently detaches from the environment and may become indifferent to family members. In addition, the family may take longer to accept the inevitable death than does the client. Although the family may not understand the anger, dealing with the resultant behavior may serve as a diversion. Denial often is exhibited by the client and family members at the same time. During depression, the family often is able to offer emotional support, which meets their needs.

A visitor comes to the nursing station and tells the nurse that a client and his relative had a fight and that the client is now lying unconscious on the floor. What is the most important action the nurse needs to take? 1 Ask the client if he is okay. 2 Call security from the room. 3 Find out if there is anyone else in the room. 4 Ask security to make sure the room is safe

4 Safety is the first priority when responding to a presumably violent situation. The nurse needs to have security enter the room to ensure it is safe. Then it can be determined if the client is okay and make sure that any other people in the room are safe

A postoperative client says to the nurse, "My neighbor, I mean the person in the next room, sings all night and keeps me awake." The neighboring client has dementia and is awaiting transfer to a nursing home. How can the nurse best handle this situation? 1 Tell the neighboring client to stop singing. 2 Close the doors to both clients' rooms at night. 3 Give the complaining client the prescribed as needed sedative. 4 Move the neighboring client to a room at the end of the hall

4 Moving the client who is singing away from the other clients diminishes the disturbance. A client with dementia will not remember instructions. It is unsafe to close the doors of clients' rooms because they need to be monitored. The use of a sedative should not be the initial intervention

The spouse of a comatose client who has severe internal bleeding refuses to allow transfusions of whole blood because they are Jehovah's Witnesses. The client does not have a Durable Power of Attorney for Healthcare. What action should the nurse take? 1 Institute the prescribed blood transfusion because the client's survival depends on volume replacement. 2 Clarify the reason why the transfusion is necessary and explain the implications if there is no transfusion. 3 Phone the health care provider for an administrative prescription to give the transfusion under these circumstances. 4 Give the spouse a treatment refusal form to sign and notify the health care provider that a court order now can be sought

4 The client is unconscious. Although the spouse can give consent, there is no legal power to refuse a treatment for the client unless previously authorized to do so by a power of attorney or a health care proxy; the court can make a decision for the client. Explanations will not be effective at this time and will not meet the client's needs. Instituting the prescribed blood transfusion and phoning the health care provider for an administrative prescription are without legal basis, and the nurse may be held liable.

A nurse assesses a client's serum electrolyte levels in the laboratory report. What electrolyte in intracellular fluid should the nurse consider most important? 1 Sodium 2 Calcium 3 Chloride 4 Potassium

4 The concentration of potassium is greater inside the cell and is important in establishing a membrane potential, a critical factor in the cell's ability to function. Sodium is the most abundant cation of the extracellular compartment, not the intracellular compartment. Calcium is the most abundant electrolyte in the body; 99% is concentrated in the teeth and bones, and only 1% is available for bodily functions. Chloride is an extracellular, not intracellular, anion.

Following a surgery on the neck, the client asks the nurse why the head of the bed is up so high. The nurse should tell the client that the high-Fowler position is preferred for what reason? 1 To avoid strain on the incision 2 To promote drainage of the wound 3 To provide stimulation for the client 4 To reduce edema at the operative site

4 This position prevents fluid accumulation in the tissue, thereby minimizing edema. This position will neither increase nor decrease strain on the suture line. Drainage from the wound will not be affected by this position. This position will not affect the degree of stimulation.

Advocacy =

Standing up for someone, other than oneself, when they are unable, or not prepared, to make a decision, or action, for themselves.

A nurse in the surgical intensive care unit is caring for a client with a large surgical incision. The nurse reviews a list of vitamins and expects that which medication will be prescribed because of its major role in wound healing? 1 Vitamin A (Aquasol A) 2 Cyanocobalamin (Cobex) 3 Phytonadione (Mephyton) 4 Ascorbic acid (Ascorbicap)

4 Vitamin C (ascorbic acid) plays a major role in wound healing . It is necessary for the maintenance and formation of collagen, the major protein of most connective tissues. Vitamin A is important for the healing process; however, vitamin C is the priority because it cements the ground substance of supportive tissue. Cyanocobalamin is a vitamin B12 preparation needed for red blood cell synthesis and a healthy nervous system. Phytonadione is vitamin K, which plays a major role in blood coagulation.

The nurse plans care for a client with a somatoform disorder based on the understanding that the disorder is: 1 A physiological response to stress 2 A conscious defense against anxiety 3 An intentional attempt to gain attention 4 An unconscious means of reducing stress

4 When emotional stress overwhelms an individual's ability to cope, the unconscious seeks to reduce stress. A conversion reaction removes the client from the stressful situation, and the conversion reaction's physical/sensory manifestation causes little or no anxiety in the individual. This lack of concern is called la belle indifference. No physiologic changes are involved with this unconscious resolution of a conflict. The conversion of anxiety to physical symptoms operates on an unconscious level.

Which nursing activities are examples of primary prevention? Select all that apply. 1 Preventing disabilities 2 Correcting dietary deficiencies 3 Establishing goals for rehabilitation 4 Assisting with immunization programs 5 t

4,5 Immunization programs prevent the occurrence of disease and are considered primary interventions. Stopping smoking prevents the occurrence of disease and is considered a primary intervention. Preventing disabilities is a tertiary intervention. Correcting dietary deficiencies is a secondary intervention. Establishing goals for rehabilitation is a tertiary intervention.

Sinus bradycardia is a dysrhythmia that proceeds normally through the conduction pathway but at a slower than usual (≤60 beats/minute) rate. Sinus bradycardia is a slower than usual (≤60 beats/minute) heart rate.

A 66-year-old female client is having cardiac diagnostic tests to determine the cause of her symptoms. In her follow-up visit to the cardiologist, she is told that she has a dysrhythmia at a rate slower than 60 beats/minute. What type of dysrhythmia did the tests reveal?

A client is receiving a cephalosporin antibiotic IV and complains of pain and irritation at the infusion site. The nurse observes erythema, swelling, and a red streak along the vessel above the IV access site. Which action should the nurse take at this time? A. Administer the medication more rapidly using the same IV site. B. Initiate an alternate site for the IV infusion of the medication. C. Notify the healthcare provider before administering the next dose. D. Give the client a PRN dose of aspirin while the medication infuses.

A cephalosporin antibiotic that is administered IV may cause vessel irritation. Rotating the infusion site minimizes the risk of thrombophlebitis, so an alternate infusion site should be initiated (B) before administering the next dose. Rapid administration (A) of intravenous cephalosporins can potentiate vessel irritation and increase the risk of thrombophlebitis. (C) is not necessary to initiate an alternative IV site. Although aspirin has antiinflammatory actions, (D) is not indicated. Correct Answer: B

A female client asks the nurse to find someone who can translate into her native language her concerns about a treatment. Which action should the nurse take? A. Explain that anyone who speaks her language can answer her questions. B. Provide a translator only in an emergency situation. C. Ask a family member or friend of the client to translate. D. Request and document the name of the certified translator.

A certified translator should be requested to ensure the exchanged information is reliable and unaltered. To adhere to legal requirements in some states, the name of the translator should be documented (D). Client information that is translated is private and protected under HIPAA rules, so (A) is not the best action. Although an emergency situation may require extenuating circumstances (B), a translator should be provided in most situations. Family members may skew information and not translate the exact information, so (C) is not preferred. Correct Answer: D

Spasms in atrial (many pwaves), blood pools-- tx with anticoagulant (warfarin), cardizem, digoxin & cardiovert if symptoms present

A fib

The nurse is assessing the nutritional status of several clients. Which client has the greatest nutritional need for additional intake of protein? A. A college-age track runner with a sprained ankle. B. A lactating woman nursing her 3-day-old infant. C. A school-aged child with Type 2 diabetes. D. An elderly man being treated for a peptic ulcer.

A lactating woman (B) has the greatest need for additional protein intake. (A, C, and D) are all conditions that require protein, but do not have the increased metabolic protein demands of lactation. Correct Answer: B

What's a tort?

A wrong committed by a person against another person or property; tried in civil court.

Ketones accumulate in the blood and urine when fat breaks down. Ketones signal a deficiency of insulin that will cause the body to start to break down stored fat for energy. Explanation: Ketones (or ketone bodies) are byproducts of fat breakdown, and they accumulate in the blood and urine. Ketones in the urine signal a deficiency of insulin and control of type 1 diabetes is deteriorating. When almost no effective insulin is available, the body starts to break down stored fat for energy

A nurse is teaching a patient recovering from diabetic ketoacidosis (DKA) about management of "sick days." The patient asks the nurse why it is important to monitor the urine for ketones. Which of the following statements is the nurse's best response?

A postoperative client will need to perform daily dressing changes after discharge. Which outcome statement best demonstrates the client's readiness to manage his wound care after discharge? The client A. asks relevant questions regarding the dressing change. B. states he will be able to complete the wound care regimen. C. demonstrates the wound care procedure correctly. D. has all the necessary supplies for wound care.

A return demonstration of a procedure (C) provides an objective assessment of the client's ability to perform a task, while (A and B) are subjective measures. (D) is important, but is less of a priority prior to discharge than the nurse's assessment of the client's ability to complete the wound care. Correct Answer: C

While instructing a male client's wife in the performance of passive range-of-motion exercises to his contracted shoulder, the nurse observes that she is holding his arm above and below the elbow. What nursing action should the nurse implement? A) Acknowledge that she is supporting the arm correctly. B) Encourage her to keep the joint covered to maintain warmth. C) Reinforce the need to grip directly under the joint for better support. D) Instruct her to grip directly over the joint for better motion.

A) Acknowledge that she is supporting the arm correctly The wife is performing the passive ROM correctly, therefore the nurse should acknowledge this fact (A). The joint that is being exercised should be uncovered (B) while the rest of the body should remain covered for warmth and privacy. (C and D) do not provide adequate support to the joint while still allowing for joint movement

An obese male client discusses with the nurse his plans to begin a long-term weight loss regimen. In addition to dietary changes, he plans to begin an intensive aerobic exercise program 3 to 4 times a week and to take stress management classes. After praising the client for his decision, which instruction is most important for the nurse to provide? A) Be sure to have a complete physical examination before beginning your planned exercise program. B) Be careful that the exercise program doesn't simply add to your stress level, making you want to eat more. C) Increased exercise helps to reduce stress, so you may not need to spend money on a stress management class. D) Make sure to monitor your weight loss regularly to provide a sense of accomplishment and motivation.

A) Be sure to have a complete physical examination before beginning your planned exercise program The most important teaching is (A), so that the client will not begin a dangerous level of exercise when he is not sufficiently fit. This might result in chest pain, a heart attack, or stroke. (B, C, and D) are important instructions, but are of less priority than (A).

The nurse plans to obtain health assessment information from a primary source. Which option is a primary source for the completion of the health assessment? A) Client. B) Healthcare provider. C) A family member. D) Previous medical records

A) Client A primary source of information for a health assessment is the client (A). (B, C, and D) are considered secondary sources about the client's health history, but other details, such as subjective data, can only be provided directly from the client.

A client with chronic renal failure selects a scrambled egg for his breakfast. What action should the nurse take? A) Commend the client for selecting a high biologic value protein. B) Remind the client that protein in the diet should be avoided. C) Suggest that the client also select orange juice, to promote absorption. D) Encourage the client to attend classes on dietary management of CRF

A) Commend the client for selecting a high biologic value protein Foods such as eggs and milk (A) are high biologic proteins which are allowed because they are complete proteins and supply the essential amino acids that are necessary for growth and cell repair. Although a low-protein diet is followed (B), some protein is essential. Orange juice is rich in potassium, and should not be encouraged (C). The client has made a good diet choice, so (D) is not necessary

After completing an assessment and determining that a client has a problem, which action should the nurse perform next? A) Determine the etiology of the problem. B) Prioritize nursing care interventions. C) Plan appropriate interventions. D) Collaborate with the client to set goals.

A) Determine the etiology of the problem Before planning care, the nurse should determine the etiology, or cause, of the problem (A), because this will help determine (B, C, and D).

The nurse is using a genogram while conducting a client's health assessment and past medical history. What information should the genogram provide? A) Genetic and familial health disorders. B) Chronic health problems. C) Reason for seeking health care. D) Undetected disorders.

A) Genetic and familial health disorders A genogram that is used during the health assessment process identifies genetic and familial health disorders (A). It may not identify the client's chronic health problems (B), so it is not a reason to seek health care (C). A genogram is not a diagnostic tool to detect disorders (D), such as those based on pathological findings or DNA.

An adult male client with a history of hypertension tells the nurse that he is tired of taking antihypertensive medications and is going to try spiritual meditation instead. What should be the nurse's first response? A) It is important that you continue your medication while learning to meditate. B) Spiritual meditation requires a time commitment of 15 to 20 minutes daily. C) Obtain your healthcare provider's permission before starting meditation. D) Complementary therapy and western medicine can be effective for you.

A) It is important that you continue your medication while learning to meditate The prolonged practice of meditation may lead to a reduced need for antihypertensive medications. However, the medications must be continued (A) while the physiologic response to meditation is monitored. (B) is not as important as continuing the medication. The healthcare provider should be informed, but permission is not required to meditate (C). Although it is true that this complimentary therapy might be effective (D), it is essential that the client continue with antihypertensive medications until the effect of meditation can be measured

During the daily nursing assessment, a client begins to cry and states that the majority of family and friends have stopped calling and visiting. What action should the nurse take? A) Listen and show interest as the client expresses these feelings. B) Reinforce that this behavior means they were not true friends. C) Ask the healthcare provider for a psychiatric consult. D) Continue with the assessment and tell the client not to worry.

A) Listen and show interest as the client expresses these feelings When a client begins to cry and express feelings, a therapeutic nursing intervention is to listen and show interest as the client expresses feelings (A). (B) is not therapeutic option and the nurse does not know the dynamics of their relationships. (C) is not indicated at this time. (D) is non-therapeutic and offers false hope

A client who is 5' 5" tall and weighs 200 pounds is scheduled for surgery the next day. What question is most important for the nurse to include during the preoperative assessment? A) What is your daily calorie consumption? B) What vitamin and mineral supplements do you take? C) Do you feel that you are overweight? D) Will a clear liquid diet be okay after surgery?

A) What is your daily calorie consumption? Vitamin and mineral supplements (B) may impact medications used during the operative period. (A and C) are appropriate questions for long-term dietary counseling. The nature of the surgery and anesthesia will determine the need for a clear liquid diet (D), rather than the client's preference

A client is to receive 10 mEq of KCl diluted in 250 ml of normal saline over 4 hours. At what rate should the nurse set the client's intravenous infusion pump? A) 13 ml/hour. B) 63 ml/hour. C) 80 ml/hour. D) 125 ml/hour

B) 63 ml/hour

Private Law?

AKA: Civil law. Most law associated with nursing. -Regulates relationships among people.

During a physical assessment, a female client begins to cry. Which action is best for the nurse to take? A. Request another nurse to complete the physical assessment. B. Ask the client to stop crying and tell the nurse what is wrong. C. Acknowledge the client's distress and tell her it is all right to cry. D. Leave the room so that the client can be alone to cry in private.

Acknowledging the client's distress and giving the client the opportunity to verbalize her distress (C) is a supportive response. (A, B, and D) are not supportive and do not facilitate the client's expression of feelings. Correct Answer: C

Integrity =

Acting according to code of ethics and standards of practice

In a nutshell, the ICN's key values of nursing are:

Advocacy, promotion of a safe environment, research, education, and participation in shaping health policy and in patient and health systems management.

Serum Potassium

After being sick for 3 days, a client with a history of diabetes mellitus is admitted to the hospital with diabetic ketoacidosis (DKA). The nurse should evaluate which diagnostic test results to prevent arrhythmias?

A client with acute hemorrhagic anemia is to receive four units of packed RBCs (red blood cells) as rapidly as possible. Which intervention is most important for the nurse to implement? A. Obtain the pre-transfusion hemoglobin level. B. Prime the tubing and prepare a blood pump set-up. C. Monitor vital signs q15 minutes for the first hour. D. Ensure the accuracy of the blood type match.

All interventions should be implemented prior to administering blood, but (D) has the highest priority. Any time blood is administered, the nurse should ensure the accuracy of the blood type match in order to prevent a possible hemolytic reaction. Correct Answer: D

ANA =

American Nursing Association

used to treat ventricular fibrillation and unstable ventricular tachycardia

Amiodarone

A client with pneumonia has a decrease in oxygen saturation from 94% to 88% while ambulating. Based on these findings, which intervention should the nurse implement first? A. Assist the ambulating client back to the bed. B. Encourage the client to ambulate to resolve pneumonia. C. Obtain a prescription for portable oxygen while ambulating. D. Move the oximetry probe from the finger to the earlobe.

An oxygen saturation below 90% indicates inadequate oxygenation. First, the client should be assisted to return to bed (A) to minimize oxygen demands. Ambulation increases aeration of the lungs to prevent pooling of respiratory secretions, but the client's activity at this time is depleting oxygen saturation of the blood, so (B) is contraindicated. Increased activity increases respiratory effort, and oxygen may be necessary to continue ambulation (C), but first the client should return to bed to rest. Oxygen saturation levels at different sites should be evaluated after the client returns to bed (D). Correct Answer: A

Acls and cpr asap!

Asystole

Sawtooth Atrial rate 250-350 Ventricular rate is steady Cardioversion, cardizem (verapamil), amiodarone

Atrial flutter

Nonmaleficence =

Avoid causing harm

Seconal 0.1 gram PRN at bedtime is prescribed to a client for rest. The scored tablets are labeled grain 1.5 per tablet. How many tablets should the nurse plan to administer? A) 0.5 tablet. B) 1 tablet. C) 1.5 tablets. D) 2 tablets.

B) 1 tablet 15 gr=1 Gm. Converting the prescribed dose of 0.1 grams to grains requires multiplying 0.1 × 15 = 1.5 grains. The tablets come in 1.5 grains, so the nurse should plan to administer 1 tablet (B).

The healthcare provider prescribes furosemide (Lasix) 15 mg IV stat. On hand is Lasix 20 mg/2 ml. How many milliliters should the nurse administer? A) 1 ml. B) 1.5 ml. C) 1.75 ml. D) 2 ml.

B) 1.5 ml

A male client being discharged with a prescription for the bronchodilator theophylline tells the nurse that he understands he is to take three doses of the medication each day. Since, at the time of discharge, timed-release capsules are not available, which dosing schedule should the nurse advise the client to follow? A) 9 a.m., 1 p.m., and 5 p.m. B) 8 a.m., 4 p.m., and midnight. C) Before breakfast, before lunch and before dinner. D) With breakfast, with lunch, and with dinner.

B) 8 a.m., 4 p.m., and midnight Theophylline should be administered on a regular around-the-clock schedule (B) to provide the best bronchodilating effect and reduce the potential for adverse effects. (A, C, and D) do not provide around-the-clock dosing. Food may alter absorption of the medication (D).

What is provision 3 of the code of ethics?

The nurse promotes, advocates for, and protects the rights, health, and safety of the patient.

What is the most important reason for starting intravenous infusions in the upper extremities rather than the lower extremities of adults? A) It is more difficult to find a superficial vein in the feet and ankles. B) A decreased flow rate could result in the formation of a thrombosis. C) A cannulated extremity is more difficult to move when the leg or foot is used. D) Veins are located deep in the feet and ankles, resulting in a more painful procedure

B) A decreased flow rate could result in the formation of a thrombosis Venous return is usually better in the upper extremities. Cannulation of the veins in the lower extremities increases the risk of thrombus formation (B) which, if dislodged, could be life-threatening. Superficial veins are often very easy (A) to find in the feet and legs. Handling a leg or foot with an IV (C) is probably not any more difficult than handling an arm or hand. Even if the nurse did believe moving a cannulated leg was more difficult, this is not the most important reason for using the upper extremities. Pain (D) is not a consideration

The nurse is assessing the nutritional status of several clients. Which client has the greatest nutritional need for additional intake of protein? A) A college-age track runner with a sprained ankle. B) A lactating woman nursing her 3-day-old infant. C) A school-aged child with Type 2 diabetes. D) An elderly man being treated for a peptic ulcer.

B) A lactating woman nursing her 3-day-old infant A lactating woman (B) has the greatest need for additional protein intake. (A, C, and D) are all conditions that require protein, but do not have the increased metabolic protein demands of lactation

On admission, a client presents a signed living will that includes a Do Not Resuscitate (DNR) prescription. When the client stops breathing, the nurse performs cardiopulmonary resuscitation (CPR) and successfully revives the client. What legal issues could be brought against the nurse? A) Assault. B) Battery. C) Malpractice. D) False imprisonment.

B) Battery Civil laws protect individual rights and include intentional torts, such as assault (an intentional threat to engage in harmful contact with another) or battery (unwanted touching). Performing any procedure against the client's wishes can potentially poise a legal issue, such as battery (B), even if the procedure is of questionable benefit to the client. (A, C, and D) are not examples against the client's request

A client who is a Jehovah's Witness is admitted to the nursing unit. Which concern should the nurse have for planning care in terms of the client's beliefs? A) Autopsy of the body is prohibited. B) Blood transfusions are forbidden. C) Alcohol use in any form is not allowed. D) A vegetarian diet must be followed

B) Blood transfusions are forbidden Blood transfusions are forbidden (B) in the Jehovah's Witness religion. Judaism prohibits (A). Buddhism forbids the use of (C) and drugs. Many of these sects are vegetarian (D), but the direct impact on nursing care is (B).

The nurse is teaching a client proper use of an inhaler. When should the client administer the inhaler-delivered medication to demonstrate correct use of the inhaler? A) Immediately after exhalation. B) During the inhalation. C) At the end of three inhalations. D) Immediately after inhalation

B) During the inhalation The client should be instructed to deliver the medication during the last part of inhalation (B). After the medication is delivered, the client should remove the mouthpiece, keeping his/her lips closed and breath held for several seconds to allow for distribution of the medication. The client should not deliver the dose as stated in (A or D), and should deliver no more than two inhalations at a time (C).

An elderly male client who is unresponsive following a cerebral vascular accident (CVA) is receiving bolus enteral feedings though a gastrostomy tube. What is the best client position for administration of the bolus tube feedings? A) Prone. B) Fowler's. C) Sims'. D) Supine.

B) Fowler's The client should be positioned in a semi-sitting (Fowler's) (B) position during feeding to decrease the occurrence of aspiration. A gastrostomy tube, known as a PEG tube, due to placement by a percutaneous endoscopic gastrostomy procedure, is inserted directly into the stomach through an incision in the abdomen for long-term administration of nutrition and hydration in the debilitated client. In (A and/or C), the client is placed on the abdomen, an unsafe position for feeding. Placing the client in (D) increases the risk of aspiration

An elderly male client who suffered a cerebral vascular accident is receiving tube feedings via a gastrostomy tube. The nurse knows that the best position for this client during administration of the feedings is A) prone. B) Fowler's. C) Sims'. D) supine

B) Fowler's The client should be positioned in a semi-sitting (Fowler's) (B) position during feeding to decrease the occurrence of aspiration. A gastrostomy tube, known as a PEG tube, due to placement by a percutaneous endoscopic gastrostomy procedure, is inserted directly into the stomach through an incision in the abdomen for long-term administration of nutrition and hydration in the debilitated client. In (A and/or C), the client is placed on the abdomen, an unsafe position for feeding. Placing the client in (D) increases the risk of aspiration

A client is receiving a cephalosporin antibiotic IV and complains of pain and irritation at the infusion site. The nurse observes erythema, swelling, and a red streak along the vessel above the IV access site. Which action should the nurse take at this time? A) Administer the medication more rapidly using the same IV site. B) Initiate an alternate site for the IV infusion of the medication. C) Notify the healthcare provider before administering the next dose. D) Give the client a PRN dose of aspirin while the medication infuses

B) Initiate an alternate site for the IV infusion of the medication A cephalosporin antibiotic that is administered IV may cause vessel irritation. Rotating the infusion site minimizes the risk of thrombophlebitis, so an alternate infusion site should be initiated (B) before administering the next dose. Rapid administration (A) of intravenous cephalosporins can potentiate vessel irritation and increase the risk of thrombophlebitis. (C) is not necessary to initiate an alternative IV site. Although aspirin has antiinflammatory actions, (D) is not indicated

A young mother of three children complains of increased anxiety during her annual physical exam. What information should the nurse obtain first? A) Sexual activity patterns. B) Nutritional history. C) Leisure activities. D) Financial stressors

B) Nutritional history Caffeine, sugars, and alcohol can lead to increased levels of anxiety, so a nutritional history (C) should be obtained first so that health teaching can be initiated if indicated. (A and C) can be used for stress management. Though (D) can be a source of anxiety, a nutritional history should be obtained first

Three days following surgery, a male client observes his colostomy for the first time. He becomes quite upset and tells the nurse that it is much bigger than he expected. What is the best response by the nurse? A) Reassure the client that he will become accustomed to the stoma appearance in time. B) Instruct the client that the stoma will become smaller when the initial swelling diminishes. C) Offer to contact a member of the local ostomy support group to help him with his concerns. D) Encourage the client to handle the stoma equipment to gain confidence with the procedure

B) Instruct the client that the stoma will become smaller when the initial swelling diminishes Postoperative swelling causes enlargement of the stoma. The nurse can teach the client that the stoma will become smaller when the swelling is diminished (B). This will help reduce the client's anxiety and promote acceptance of the colostomy. (A) does not provide helpful teaching or support. (C) is a useful action, and may be taken after the nurse provides pertinent teaching. The client is not yet demonstrating readiness to learn colostomy care (D).

Examination of a client complaining of itching on his right arm reveals a rash made up of multiple flat areas of redness ranging from pinpoint to 0.5 cm in diameter. How should the nurse record this finding? A) Multiple vesicular areas surrounded by redness, ranging in size from 1 mm to 0.5 cm. B) Localized red rash comprised of flat areas, pinpoint to 0.5 cm in diameter. C) Several areas of red, papular lesions from pinpoint to 0.5 cm in size. D) Localized petechial areas, ranging in size from pinpoint to 0.5 cm in diameter.

B) Localized red rash comprised of flat areas, pinpoint to 0.5 cm in diameter Macules are localized flat skin discolorations less than 1 cm in diameter. However, when recording such a finding the nurse should describe the appearance (B) rather than simply naming the condition. (A) identifies vesicles -- fluid filled blisters -- an incorrect description given the symptoms listed. (C) identifies papules -- solid elevated lesions, again not correctly identifying the symptoms. (D) identifies petechiae -- pinpoint red to purple skin discolorations that do not itch, again an incorrect identification

A female client with a nasogastric tube attached to low suction states that she is nauseated. The nurse assesses that there has been no drainage through the nasogastric tube in the last two hours. What action should the nurse take first? A) Irrigate the nasogastric tube with sterile normal saline. B) Reposition the client on her side. C) Advance the nasogastric tube an additional five centimeters. D) Administer an intravenous antiemetic prescribed for PRN use.

B) Reposition the client on her side The immediate priority is to determine if the tube is functioning correctly, which would then relieve the client's nausea. The least invasive intervention, (B), should be attempted first, followed by (A and C), unless either of these interventions is contraindicated. If these measures are unsuccessful, the client may require an antiemetic (D).

When assisting an 82-year-old client to ambulate, it is important for the nurse to realize that the center of gravity for an elderly person is the A) Arms. B) Upper torso. C) Head. D) Feet

B) Upper torso The center of gravity for adults is the hips. However, as the person grows older, a stooped posture is common because of the changes from osteoporosis and normal bone degeneration, and the knees, hips, and elbows flex. This stooped posture results in the upper torso (B) becoming the center of gravity for older persons. Although (A) is a part, or an extension of the upper torso, this is not the best and most complete answer.

In developing a plan of care for a client with dementia, the nurse should remember that confusion in the elderly A) is to be expected, and progresses with age. B) often follows relocation to new surroundings. C) is a result of irreversible brain pathology. D) can be prevented with adequate sleep

B) often follows relocation to new surroundings Relocation (B) often results in confusion among elderly clients--moving is stressful for anyone. (A) is a stereotypical judgment. Stress in the elderly often manifests itself as confusion, so (C) is wrong. Adequate sleep is not a prevention (D) for confusion

What is provision 4 of the code of ethics?

The nurse has authority, accountability, and responsibility for nursing practice' makes decisions; and takes action consistent with the obligation to promote health and to provide optimal care.

The healthcare provider prescribes 1,000 ml of Ringer's Lactate with 30 Units of Pitocin to run in over 4 hours for a client who has just delivered a 10 pound infant by cesarean section. The tubing has been changed to a 20 gtt/ml administration set. The nurse plans to set the flow rate at how many gtt/min? A) 42 gtt/min. B) 83 gtt/min. C) 125 gtt/min. D) 250 gtt/min

B. 83 gtt/min

After completing an assessment and determining that a client has a problem, which action should the nurse perform next? A. Determine the etiology of the problem. B. Prioritize nursing care interventions. C. Plan appropriate interventions. D. Collaborate with the client to set goals.

Before planning care, the nurse should determine the etiology, or cause, of the problem (A), because this will help determine (B, C, and D). Correct Answer: A

A client who is a Jehovah's Witness is admitted to the nursing unit. Which concern should the nurse have for planning care in terms of the client's beliefs? A. Autopsy of the body is prohibited. B. Blood transfusions are forbidden. C. Alcohol use in any form is not allowed. D. A vegetarian diet must be followed.

Blood transfusions are forbidden (B) in the Jehovah's Witness religion. Judaism prohibits (A). Buddhism forbids the use of (C) and drugs. Many of these sects are vegetarian (D), but the direct impact on nursing care is (B). Correct Answer: B

Which assessment data would provide the most accurate determination of proper placement of a nasogastric tube? A. Aspirating gastric contents to assure a pH value of 4 or less. B. Hearing air pass in the stomach after injecting air into the tubing. C. Examining a chest x-ray obtained after the tubing was inserted. D. Checking the remaining length of tubing to ensure that the correct length was inserted.

Both (A and B) are methods used to determine proper placement of the NG tubing. However, the best indicator that the tubing is properly placed is (C). (D) is not an indicator of proper placement. Correct Answer: C

The healthcare provider prescribes the diuretic metolazone (Zaroxolyn) 7.5 mg PO. Zaroxolyn is available in 5 mg tablets. How much should the nurse plan to administer? A) ½ tablet. B) 1 tablet. C) 1½ tablets. D) 2 tablets.

C) 1½ tablets

Desmopressin Why?

The nurse is caring for a client with diabetes insipidus. The nurse should anticipate the administration of:

Which response by a client with a nursing diagnosis of Spiritual distress, indicates to the nurse that a desired outcome measure has been met? A) Expresses concern about the meaning and importance of life B) Remains angry at God for the continuation of the illness. C) Accepts that punishment from God is not related to illness. D) Refuses to participate in religious rituals that have no meaning.

C) Accepts that punishment from God is not related to illness Acceptance that she is not being punished by God indicates a desired outcome (C) for some degree of resolution of spiritual distress. (A, B, and D) do not support the concept of grief, loss, and cultural/spiritual acceptance.

During a physical assessment, a female client begins to cry. Which action is best for the nurse to take? A) Request another nurse to complete the physical assessment. B) Ask the client to stop crying and tell the nurse what is wrong. C) Acknowledge the client's distress and tell her it is all right to cry. D) Leave the room so that the client can be alone to cry in private.

C) Acknowledge the client's distress and tell her it is all right to cry Acknowledging the client's distress and giving the client the opportunity to verbalize her distress (C) is a supportive response. (A, B, and D) are not supportive and do not facilitate the client's expression of feelings

A hospitalized male client is receiving nasogastric tube feedings via a small-bore tube and a continuous pump infusion. He reports that he had a bad bout of severe coughing a few minutes ago, but feels fine now. What action is best for the nurse to take? A) Record the coughing incident. No further action is required at this time. B) Stop the feeding, explain to the family why it is being stopped, and notify the healthcare provider. C) After clearing the tube with 30 ml of air, check the pH of fluid withdrawn from the tube. D) Inject 30 ml of air into the tube while auscultating the epigastrium for gurgling.

C) After clearing the tube with 30 ml of air, check the pH of fluid withdrawn from the tube Coughing, vomiting, and suctioning can precipitate displacement of the tip of the small bore feeding tube upward into the esophagus, placing the client at increased risk for aspiration. Checking the sample of fluid withdrawn from the tube (after clearing the tube with 30 ml of air) for acidic (stomach) or alkaline (intestine) values is a more sensitive method for these tubes, and the nurse should assess tube placement in this way prior to taking any other action (C). (A and B) are not indicated. The auscultating method (D) has been found to be unreliable for small-bore feeding tubes.

During a visit to the outpatient clinic, the nurse assesses a client with severe osteoarthritis using a goniometer. Which finding should the nurse expect to measure? A) Adequate venous blood flow to the lower extremities. B) Estimated amount of body fat by an underarm skinfold. C) Degree of flexion and extension of the client's knee joint. D) Change in the circumference of the joint in centimeters

C) Degree of flexion and extension of the client's knee joint The goniometer is a two-piece ruler that is jointed in the middle with a protractor-type measuring device that is placed over a joint as the individual extends or flexes the joint to measure the degrees of flexion and extension on the protractor (C). A doppler is used to measure blood flow (A). Calipers are used to measure body fat (B). A tape measure is used to measure circumference of body parts (D).

A client who has been NPO for 3 days is receiving an infusion of D5W 0.45 normal saline (NS) with potassium chloride (KCl) 20 mEq at 83 ml/hour. The client's eight-hour urine output is 400 ml, blood urea nitrogen (BUN) is 15 mg/dl, lungs are clear bilaterally, serum glucose is 120 mg/dl, and the serum potassium is 3.7 mEq/L. Which action is most important for the nurse to implement? A) Notify healthcare provider and request to change the IV infusion to hypertonic D10W. B) Decrease in the infusion rate of the current IV and report to the healthcare provider. C) Document in the medical record that these normal findings are expected outcomes. D) Obtain potassium chloride 20 mEq in anticipation of a prescription to add to present IV.

C) Document in the medical record that these normal findings are expected outcomes The results are all within normal range.(C) No changes are needed. (A,B, and D)

Which assessment data would provide the most accurate determination of proper placement of a nasogastric tube? A) Aspirating gastric contents to assure a pH value of 4 or less. B) Hearing air pass in the stomach after injecting air into the tubing. C) Examining a chest x-ray obtained after the tubing was inserted. D) Checking the remaining length of tubing to ensure that the correct length was inserted.

C) Examining a chest x-ray obtained after the tubing was inserted Both (A and B) are methods used to determine proper placement of the NG tubing. However, the best indicator that the tubing is properly placed is (C). (D) is not an indicator of proper placement

A resident in a skilled nursing facility for short-term rehabilitation after a hip replacement tells the nurse, "I don't want any more blood taken for those useless tests." Which narrative documentation should the nurse enter in the client's medical record? A) Healthcare provider notified of failure to collect specimens for prescribed blood studies. B) Blood specimens not collected because client no longer wants blood tests performed. C) Healthcare provider notified of client's refusal to have blood specimens collected for testing. D) Client irritable, uncooperative, and refuses to have blood collected. Healthcare provider notified

C) Healthcare provider notified of client's refusal to have blood specimens collected for testing When a client refuses a treatment, the exact words of the client regarding the client's refusal of care should be documented in a narrative format (C). (A, B, and D) do not address the concepts of informatics and legal issues

The nurse is instructing a client with high cholesterol about diet and life style modification. What comment from the client indicates that the teaching has been effective? A) If I exercise at least two times weekly for one hour, I will lower my cholesterol. B) I need to avoid eating proteins, including red meat. C) I will limit my intake of beef to 4 ounces per week. D) My blood level of low density lipoproteins needs to increase.

C) I will limit my intake of beef to 4 ounces per week Limiting saturated fat from animal food sources to no more than 4 ounces per week (C) is an important diet modification for lowering cholesterol. To be effective in reducing cholesterol, the client should exercise 30 minutes per day, or at least 4 to 6 times per week (A). Red meat and all proteins do not need to be eliminated (B) to lower cholesterol, but should be restricted to lean cuts of red meat and smaller portions (2-ounce servings). The low density lipoproteins (D) need to decrease rather than increase

The nurse is caring for a client who is receiving 24-hour total parenteral nutrition (TPN) via a central line at 54 ml/hr. When initially assessing the client, the nurse notes that the TPN solution has run out and the next TPN solution is not available. What immediate action should the nurse take? A) Infuse normal saline at a keep vein open rate. B) Discontinue the IV and flush the port with heparin. C) Infuse 10 percent dextrose and water at 54 ml/hr D) Obtain a stat blood glucose level and notify the healthcare provider.

C) Infuse 10 percent dextrose and water at 54 ml/hr TPN is discontinued gradually to allow the client to adjust to decreased levels of glucose. Administering 10% dextrose in water at the prescribed rate (C) will keep the client from experiencing hypoglycemia until the next TPN solution is available. The client could experience a hypoglycemic reaction if the current level of glucose (A) is not maintained or if the TPN is discontinued abruptly (B). There is no reason to obtain a stat blood glucose level (D) and the healthcare provider cannot do anything about this situation

Which action is most important for the nurse to implement when donning sterile gloves? A) Maintain thumb at a ninety degree angle. B) Hold hands with fingers down while gloving. C) Keep gloved hands above the elbows. D) Put the glove on the dominant hand first.

C) Keep gloved hands above the elbows Gloved hands held below waist level are considered unsterile (C). (A and B) are not essential to maintaining asepsis. While it may be helpful to put the glove on the dominant hand first, it is not necessary to ensure asepsis (D).

When conducting an admission assessment, the nurse should ask the client about the use of complimentary healing practices. Which statement is accurate regarding the use of these practices? A) Complimentary healing practices interfere with the efficacy of the medical model of treatment. B) Conventional medications are likely to interact with folk remedies and cause adverse effects. C) Many complimentary healing practices can be used in conjunction with conventional practices. D) Conventional medical practices will ultimately replace the use of complimentary healing practices.

C) Many complimentary healing practices can be used in conjunction with conventional practices Conventional approaches to health care can be depersonalizing and often fail to take into consideration all aspects of an individual, including body, mind, and spirit. Often complimentary healing practices can be used in conjunction with conventional medical practices (C), rather than interfering (A) with conventional practices, causing adverse effects (B), or replacing conventional medical care (D).

The nurse assigns a UAP to obtain vital signs from a very anxious client. What instructions should the nurse give the UAP? A) Remain calm with the client and record abnormal results in the chart. B) Notify the medication nurse immediately if the pulse or blood pressure is low. C) Report the results of the vital signs to the nurse. D) Reassure the client that the vital signs are normal.

C) Report the results of the vital signs to the nurse. Interpretation of vital signs is the responsibility of the nurse, so the UAP should report vital sign measurements to the nurse (C). (A, B, and D) require the UAP to interpret the vital signs, which is beyond the scope of the UAP's authority

An unlicensed assistive personnel (UAP) places a client in a left lateral position prior to administering a soap suds enema. Which instruction should the nurse provide the UAP? A) Position the client on the right side of the bed in reverse Trendelenburg. B) Fill the enema container with 1000 ml of warm water and 5 ml of castile soap. C) Reposition in a Sim's position with the client's weight on the anterior ilium. D) Raise the side rails on both sides of the bed and elevate the bed to waist level.

C) Reposition in a Sim's position with the client's weight on the anterior ilium The left sided Sims' position allows the enema solution to follow the anatomical course of the intestines and allows the best overall results, so the UAP should reposition the client in the Sims' position, which distributes the client's weight to the anterior ilium (C). (A) is inaccurate. (B and D) should be implemented once the client is positioned

The nurse is evaluating client learning about a low-sodium diet. Selection of which meal would indicate to the nurse that this client understands the dietary restrictions? A) Tossed salad, low-sodium dressing, bacon and tomato sandwich. B) New England clam chowder, no-salt crackers, fresh fruit salad. C) Skim milk, turkey salad, roll, and vanilla ice cream. D) Macaroni and cheese, diet Coke, a slice of cherry pie.

C) Skim milk, turkey salad, roll, and vanilla ice cream Skim milk, turkey, bread, and ice cream (C), while containing some sodium, are considered low-sodium foods. Bacon (A), canned soups (B), especially those with seafood, hard cheeses, macaroni, and most diet drinks (D) are very high in sodium

A postoperative client will need to perform daily dressing changes after discharge. Which outcome statement best demonstrates the client's readiness to manage his wound care after discharge? The client A) asks relevant questions regarding the dressing change. B) states he will be able to complete the wound care regimen. C) demonstrates the wound care procedure correctly. D) has all the necessary supplies for wound care.

C) demonstrates the wound care procedure correctly A return demonstration of a procedure (C) provides an objective assessment of the client's ability to perform a task, while (A and B) are subjective measures. (D) is important, but is less of a priority prior to discharge than the nurse's assessment of the client's ability to complete the wound care

A young mother of three children complains of increased anxiety during her annual physical exam. What information should the nurse obtain first? A. Sexual activity patterns. B. Nutritional history. C. Leisure activities. D. Financial stressors.

Caffeine, sugars, and alcohol can lead to increased levels of anxiety, so a nutritional history (C) should be obtained first so that health teaching can be initiated if indicated. (A and C) can be used for stress management. Though (D) can be a source of anxiety, a nutritional history should be obtained first. Correct Answer: B

Florence Nightingale did many significant things in her lifetime, some of those include:

Challenged prejudices against women. Elevated the status of nurses Established the first "proper" training of nurses. *Based nursing practice on evidence.* Helped distinguish nursing from medicine.

The nurse notices that the Hispanic parents of a toddler who returns from surgery offer the child only the broth that comes on the clear liquid tray. Other liquids, including gelatin, popsicles, and juices, remain untouched. What explanation is most appropriate for this behavior? A. The belief is held that the "evil eye" enters the child if anything cold is ingested. B. After surgery the child probably has refused all foods except broth. C. Eating broth strengthens the child's innate energy called "chi." D. Hot remedies restore balance after surgery, which is considered a "cold" condition.

Common parental practices and health beliefs among Hispanic, Chinese, Filipino, and Arab cultures classify diseases, areas of the body, and illnesses as "hot" or "cold" and must be balanced to maintain health and prevent illness. The perception that surgery is a "cold" condition implies that only "hot" remedies, such as soup, should be used to restore the healthy balance within the body, so (D) is the correct interpretation. (A, B, and C) are not correct interpretations of the noted behavior. "Chi" is a Chinese belief that an innate energy enters and leaves the body via certain locations and pathways and maintains health. The "evil eye," or "mal ojo," is believed by many cultures to be related to the balance of health and illness but is unrelated to dietary practice. Correct Answer: D

When conducting an admission assessment, the nurse should ask the client about the use of complimentary healing practices. Which statement is accurate regarding the use of these practices? A. Complimentary healing practices interfere with the efficacy of the medical model of treatment. B. Conventional medications are likely to interact with folk remedies and cause adverse effects. C. Many complimentary healing practices can be used in conjunction with conventional practices. D. Conventional medical practices will ultimately replace the use of complimentary healing practices.

Conventional approaches to health care can be depersonalizing and often fail to take into consideration all aspects of an individual, including body, mind, and spirit. Often complimentary healing practices can be used in conjunction with conventional medical practices (C), rather than interfering (A) with conventional practices, causing adverse effects (B), or replacing conventional medical care (D). Correct Answer: C

The nursing process is the _____________ for all nursing activities.

Framework

A hospitalized male client is receiving nasogastric tube feedings via a small-bore tube and a continuous pump infusion. He reports that he had a bad bout of severe coughing a few minutes ago, but feels fine now. What action is best for the nurse to take? A. Record the coughing incident. No further action is required at this time. B. Stop the feeding, explain to the family why it is being stopped, and notify the healthcare provider. C. After clearing the tube with 30 ml of air, check the pH of fluid withdrawn from the tube. D. Inject 30 ml of air into the tube while auscultating the epigastrium for gurgling.

Coughing, vomiting, and suctioning can precipitate displacement of the tip of the small bore feeding tube upward into the esophagus, placing the client at increased risk for aspiration. Checking the sample of fluid withdrawn from the tube (after clearing the tube with 30 ml of air) for acidic (stomach) or alkaline (intestine) values is a more sensitive method for these tubes, and the nurse should assess tube placement in this way prior to taking any other action (C). (A and B) are not indicated. The auscultating method (D) has been found to be unreliable for small-bore feeding tubes. Correct Answer: C

client expresses concern about being exposed to radiation therapy because it can cause cancer. What should the nurse emphasize when informing the client about exposure to radiation? 1 The dosage is kept at a minimum. 2 Only a small part of the body is irradiated. 3 The client's physical condition is not a risk factor. 4 Nutritional environment of the affected cells is a risk factor.

Current radiation therapy accurately targets malignant lesions with pinpoint precision, minimizing the detrimental effects of radiation to healthy tissue. The dose is not as significant as the extent of tissue being irradiated. When radiation therapy is prescribed, the health care provider takes into consideration the ability of the client to tolerate the therapy, determining that the benefit outweighs the risk. Nutritional environment of the affected cells does not influence radiation's effect.

The nurse mixes 50 mg of Nipride in 250 ml of D5W and plans to administer the solution at a rate of 5 mcg/kg/min to a client weighing 182 pounds. Using a drip factor of 60 gtt/ml, how many drops per minute should the client receive? A) 31 gtt/min. B) 62 gtt/min. C) 93 gtt/min. D) 124 gtt/min

D) 124 gtt/min

A client with acute hemorrhagic anemia is to receive four units of packed RBCs (red blood cells) as rapidly as possible. Which intervention is most important for the nurse to implement? A) Obtain the pre-transfusion hemoglobin level. B) Prime the tubing and prepare a blood pump set-up. C) Monitor vital signs q15 minutes for the first hour. D) Ensure the accuracy of the blood type match.

D) Ensure the accuracy of the blood type match All interventions should be implemented prior to administering blood, but (D) has the highest priority. Any time blood is administered, the nurse should ensure the accuracy of the blood type match in order to prevent a possible hemolytic reaction

The nurse notices that the Hispanic parents of a toddler who returns from surgery offer the child only the broth that comes on the clear liquid tray. Other liquids, including gelatin, popsicles, and juices, remain untouched. What explanation is most appropriate for this behavior? A) The belief is held that the "evil eye" enters the child if anything cold is ingested. B) After surgery the child probably has refused all foods except broth. C) Eating broth strengthens the child's innate energy called "chi." D) Hot remedies restore balance after surgery, which is considered a "cold" condition.

D) Hot remedies restore balance after surgery, which is considered a "cold" condition Common parental practices and health beliefs among Hispanic, Chinese, Filipino, and Arab cultures classify diseases, areas of the body, and illnesses as "hot" or "cold" and must be balanced to maintain health and prevent illness. The perception that surgery is a "cold" condition implies that only "hot" remedies, such as soup, should be used to restore the healthy balance within the body, so (D) is the correct interpretation. (A, B, and C) are not correct interpretations of the noted behavior. "Chi" is a Chinese belief that an innate energy enters and leaves the body via certain locations and pathways and maintains health. The "evil eye," or "mal ojo," is believed by many cultures to be related to the balance of health and illness but is unrelated to dietary practice.

At the beginning of the shift, the nurse assesses a client who is admitted from the post-anesthesia care unit (PACU). When should the nurse document the client's findings? A) At the beginning, middle, and end of the shift. B) After client priorities are identified for the development of the nursing care plan. C) At the end of the shift so full attention can be given to the client's needs. D) Immediately after the assessments are completed

D) Immediately after the assessments are completed Documentation should occur immediately after any component of the nursing process, so assessments should be entered in the client's medical record as readily as findings are obtained (D). (A, B, and C) do not address the concepts of legal recommendations for information management and informatics.

The UAPs working on a chronic neuro unit ask the nurse to help them determine the safest way to transfer an elderly client with left-sided weakness from the bed to the chair. What method describes the correct transfer procedure for this client? A) Place the chair at a right angle to the bed on the client's left side before moving. B) Assist the client to a standing position, then place the right hand on the armrest. C) Have the client place the left foot next to the chair and pivot to the left before sitting. D) Move the chair parallel to the right side of the bed, and stand the client on the right foot

D) Move the chair parallel to the right side of the bed, and stand the client on the right foot (D) uses the client's stronger side, the right side, for weight-bearing during the transfer, and is the safest approach to take. (A, B, and C) are unsafe methods of transfer and include the use of poor body mechanics by the caregiver.

An elderly resident of a long-term care facility is no longer able to perform self-care and is becoming progressively weaker. The resident previously requested that no resuscitative efforts be performed, and the family requests hospice care. What action should the nurse implement first? A) Reaffirm the client's desire for no resuscitative efforts. B) Transfer the client to a hospice inpatient facility. C) Prepare the family for the client's impending death. D) Notify the healthcare provider of the family's request.

D) Notify the healthcare provider of the family's request The nurse should first communicate with the healthcare provider (D). Hospice care is provided for clients with a limited life expectancy, which must be identified by the healthcare provider. (A) is not necessary at this time. Once the healthcare provider supports the transfer to hospice care, the nurse can collaborate with the hospice staff and healthcare provider to determine when (B and C) should be implemented

An older client who is a resident in a long term care facility has been bedridden for a week. Which finding should the nurse identify as a client risk factor for pressure ulcers? A) Generalized dry skin. B) Localized dry skin on lower extremities. C) Red flush over entire skin surface. D) Rashes in the axillary, groin, and skin fold regions

D) Rashes in the axillary, groin, and skin fold regions Immobility, constant contact with bed clothing, and excessive heat and moisture in areas where air flow is limited contributes to bacterial and fungal growth, which increases the risk for rashes (D), skin breakdown, and the development of pressure ulcers. (A, B, and C) do not address the concepts of inflammation and tissue integrity

The nurse is performing nasotracheal suctioning. After suctioning the client's trachea for fifteen seconds, large amounts of thick yellow secretions return. What action should the nurse implement next? A) Encourage the client to cough to help loosen secretions. B) Advise the client to increase the intake of oral fluids. C) Rotate the suction catheter to obtain any remaining secretions. D) Re-oxygenate the client before attempting to suction again.

D) Re-oxygenate the client before attempting to suction again Suctioning should not be continued for longer than ten to fifteen seconds, since the client's oxygenation is compromised during this time (D). (A, B, and C) may be performed after the client is re-oxygenated and additional suctioning is performed.

subset of type 2 due to hormone release from placenta resist insulin 2nd & 3rd Trimester, give glucose challenge, dx if over 126

Gestational Diabetes

A female client asks the nurse to find someone who can translate into her native language her concerns about a treatment. Which action should the nurse take? A) Explain that anyone who speaks her language can answer her questions. B) Provide a translator only in an emergency situation. C) Ask a family member or friend of the client to translate. D) Request and document the name of the certified translator.

D) Request and document the name of the certified translator A certified translator should be requested to ensure the exchanged information is reliable and unaltered. To adhere to legal requirements in some states, the name of the translator should be documented (D). Client information that is translated is private and protected under HIPAA rules, so (A) is not the best action. Although an emergency situation may require extenuating circumstances (B), a translator should be provided in most situations. Family members may skew information and not translate the exact information, so (C) is not preferred.

An Arab-American woman, who is a devout traditional Muslim, lives with her married son's family, which includes several adult children and their children. What is the best plan to obtain consent for surgery for this client? A) Obtain an interpreter to explain the procedure to the client. B) Encourage the client to make her own decision regarding surgery. C) Ask the family members to provide an interpretation of the surgeon's explanation to the client. D) Tell the surgeon that the son will decide after explanation of the proposed surgery is provided.

D) Tell the surgeon that the son will decide after explanation of the proposed surgery is provided Traditional Muslim women live in a patriarchal family where decisions are made by men. Most likely, the son will make the decision for his mother, so (D) provides the surgeon with culturally sensitive information. (A) may be necessary if a language barrier exists, but the son is the patriarch in the client's family at this time. It is culturally insensitive to encourage the woman to go against her religious and cultural worldview, as in (B). Family members are more likely to misinterpret medical information, but the son should be the primary decision-maker for his mother (C).

Which nutritional assessment data should the nurse collect to best reflect total muscle mass in an adolescent? A) Height in inches or centimeters. B) Weight in kilograms or pounds. C) Triceps skin fold thickness. D) Upper arm circumference.

D) Upper arm circumference Upper arm circumference (D) is an indirect measure of muscle mass. (A and B) do not distinguish between fat (adipose) and muscularity. (C) is a measure of body fat.

A male client tells the nurse that he does not know where he is or what year it is. What data should the nurse document that is most accurate? A) demonstrates loss of remote memory. B) exhibits expressive dysphasia. C) has a diminished attention span. D) is disoriented to place and time.

D) is disoriented to place and time The client is exhibiting disorientation (D). (A) refers to memory of the distant past. The client is able to express himself without difficulty (B), and does not demonstrate a diminished attention span (C).

Justice =

Give each his or her due and act fairly

The duration of regular insulin is 4 to 6 hours; 3 to 5 hours is the duration for rapid-acting insulin such as Novolog. The duration of NPH insulin is 12 to 16 hours. The duration of Lantus insulin is 24 hours Humalog is a rapid-acting insulin. NPH is an intermediate-acting insulin. A short-acting insulin is Humulin-R. An example of a long-acting insulin is Glargine (Lantus)

Duration of Insulin is:

What is provision 5 of the code of ethics?

The nurse owes the same duties to self as to others, including the responsibility to promote health and safety, preserve wholeness of character, and integrity, maintain competence, and continue personal and professional growth.

During the initial morning assessment, a male client denies dysuria but reports that his urine appears dark amber. Which intervention should the nurse implement? A. Provide additional coffee on the client's breakfast tray. B. Exchange the client's grape juice for cranberry juice. C. Bring the client additional fruit at mid-morning. D. Encourage additional oral intake of juices and water.

Dark amber urine is characteristic of fluid volume deficit, and the client should be encouraged to increase fluid intake (D). Caffeine, however, is a diuretic (A), and may worsen the fluid volume deficit. Any type of juice will be beneficial (B), since the client is not dysuric, a sign of an urinary tract infection. The client needs to restore fluid volume more than solid foods (C). Correct Answer: D

Symptoms + BG >=200ml/dl at any time w/o regard to meal 2hr post-load glucose >200 (oral gluc test) fasting >126

Diagnostic Criteria

An African-American grandmother tells the nurse that her 4-year-old grandson is suffering with "miseries." Based on this statement, which focused assessment should the nurse conduct? A. Inquire about the source and type of pain. B. Examine the nose for congestion and discharge. C. Take vital signs for temperature elevation. D. Explore the abdominal area for distension.

Different cultural groups often have their own terms for health conditions. African-American clients may refer to pain as "the miseries. " Based on understanding this term, the nurse should conduct a focused assessment on the source and type of pain (A). (B, C, and D) are important, but do not focus on "miseries" (pain). Correct Answer: A

The nurse notices that the mother a 9-year-old Vietnamese child always looks at the floor when she talks to the nurse. What action should the nurse take? A. Talk directly to the child instead of the mother. B. Continue asking the mother questions about the child. C. Ask another nurse to interview the mother now. D. Tell the mother politely to look at you when answering.

Eye contact is a culturally-influenced form of non-verbal communication. In some non-Western cultures, such as the Vietnamese culture, a client or family member may avoid eye contact as a form of respect, so the nurse should continue to ask the mother questions about the child (B). (A, C, and D) are not indicated. Correct Answer: B

What is healthy people 2020?

Federal government indicative. Sort of like "Guidelines" for the US health standards.

Place each step of the nursing process in the order that it should be used. Correct 1. Obtain client's nursing history. Correct 2. State client's nursing needs. Correct 3. Identify goals for care. Correct 4. Develop a plan of care. Correct 5. Implement nursing interventions.

First the nurse should gather data. Based on the data, the client's needs are assessed. After the needs have been determined, the goals for care are established. The next step is planning care based on the knowledge gained from the previous steps. Implementation follows the development of the plan of care.

When evaluating a client's plan of care, the nurse determines that a desired outcome was not achieved. Which action will the nurse implement first? A. Establish a new nursing diagnosis. B. Note which actions were not implemented. C. Add additional nursing orders to the plan. D. Collaborate with the healthcare provider to make changes.

First, the nurse reviews which actions in the original plan were not implemented (B) in order to determine why the original plan did not produce the desired outcome. Appropriate revisions can then be made, which may include revising the expected outcome, or identifying a new nursing diagnosis (A). (C) may be needed if the nursing actions were unsuccessful, or were unable to be implemented. (D) other members of the healthcare team may be necessary to collaborate changes once the nurse determines why the original plan did not produce the desired outcome. Correct Answer: B

Who is the founder behind the profession of nursing?

Florence Nightingale

A client with chronic renal failure selects a scrambled egg for his breakfast. What action should the nurse take? A. Commend the client for selecting a high biologic value protein. B. Remind the client that protein in the diet should be avoided. C. Suggest that the client also select orange juice, to promote absorption. D. Encourage the client to attend classes on dietary management of CRF.

Foods such as eggs and milk (A) are high biologic proteins which are allowed because they are complete proteins and supply the essential amino acids that are necessary for growth and cell repair. Although a low-protein diet is followed (B), some protein is essential. Orange juice is rich in potassium, and should not be encouraged (C). The client has made a good diet choice, so (D) is not necessary. Correct Answer: A

Which action is most important for the nurse to implement when donning sterile gloves? A. Maintain thumb at a ninety degree angle. B. Hold hands with fingers down while gloving. C. Keep gloved hands above the elbows. D. Put the glove on the dominant hand first.

Gloved hands held below waist level are considered unsterile (C). (A and B) are not essential to maintaining asepsis. While it may be helpful to put the glove on the dominant hand first, it is not necessary to ensure asepsis (D). Correct Answer: C

Hypoglycemia

Glucagon is used primarily to treat a patient with

HIPAA =

Health Insurance Portability and Accountability Act

A client who had abdominal surgery is receiving patient-controlled analgesia (PCA) intravenously to manage pain. The pump is programmed to deliver a basal dose and bolus doses that can be accessed by the client with a lock-out time frame of 10 minutes. The nurse assesses use of the pump during the last hour and identifies that the client attempted to self-administer the analgesic 10 times. Further assessment reveals that the client is experiencing pain still. What should the nurse do first? 1 Monitor the client's pain level for another hour. 2 Determine the integrity of the intravenous delivery system. 3 Reprogram the pump to deliver a bolus dose every eight minutes. 4 Arrange for the client to be evaluated by the health care provider.

Initially, integrity of the intravenous system should be verified to ensure that the client is receiving medication. The intravenous tubing may be kinked or compressed, or the catheter may be dislodged. Continued monitoring will result in the client experiencing unnecessary pain. The nurse may not reprogram the pump to deliver larger or more frequent doses of medication without a health care provider's prescription. The health care provider should be notified if the system is intact and the client is not obtaining relief from pain. The prescription may have to be revised; the basal dose may be increased, the length of the delay may be reduced, or another medication or mode of delivery may be prescribed.

Lente, NPH Onset: 2-4 hrs Duration: 16-20 hours Peak: 4-12 hours

Intermediate

ICN =

International Council for Nurses

blood seeps & protein leaks out, leads to blindness (diabetic retinapothy)

Microvascualr Angiopathy

The nurse assigns a UAP to obtain vital signs from a very anxious client. What instructions should the nurse give the UAP? A. Remain calm with the client and record abnormal results in the chart. B. Notify the medication nurse immediately if the pulse or blood pressure is low. C. Report the results of the vital signs to the nurse. D. Reassure the client that the vital signs are normal.

Interpretation of vital signs is the responsibility of the nurse, so the UAP should report vital sign measurements to the nurse (C). (A, B, and D) require the UAP to interpret the vital signs, which is beyond the scope of the UAP's authority. Correct Answer: C

Fidelity =

Keeping promises; "Integrity"

The nurse is instructing a client with high cholesterol about diet and life style modification. What comment from the client indicates that the teaching has been effective? A. If I exercise at least two times weekly for one hour, I will lower my cholesterol. B. I need to avoid eating proteins, including red meat. C. I will limit my intake of beef to 4 ounces per week. D. My blood level of low density lipoproteins needs to increase.

Limiting saturated fat from animal food sources to no more than 4 ounces per week (C) is an important diet modification for lowering cholesterol. To be effective in reducing cholesterol, the client should exercise 30 minutes per day, or at least 4 to 6 times per week (A). Red meat and all proteins do not need to be eliminated (B) to lower cholesterol, but should be restricted to lean cuts of red meat and smaller portions (2-ounce servings). The low density lipoproteins (D) need to decrease rather than increase. Correct Answer: C

Ultra Lente, Glargine (LANTUS) Onset: 1 hr Duration: 24 hrs NO PEAK

Long Acting

When to use? Morphine, O2, nitroglycerin, aspirin

MONA

Larger vessel damage

Macroangiopathy

clots build up, accelorates athro sclerosis, which can lead to myocardial infarction

Macrovascualr Angiopathy

Examination of a client complaining of itching on his right arm reveals a rash made up of multiple flat areas of redness ranging from pinpoint to 0.5 cm in diameter. How should the nurse record this finding? A. Multiple vesicular areas surrounded by redness, ranging in size from 1 mm to 0.5 cm. B. Localized red rash comprised of flat areas, pinpoint to 0.5 cm in diameter. C. Several areas of red, papular lesions from pinpoint to 0.5 cm in size. D. Localized petechial areas, ranging in size from pinpoint to 0.5 cm in diameter.

Macules are localized flat skin discolorations less than 1 cm in diameter. However, when recording such a finding the nurse should describe the appearance (B) rather than simply naming the condition. (A) identifies vesicles -- fluid filled blisters -- an incorrect description given the symptoms listed. (C) identifies papules -- solid elevated lesions, again not correctly identifying the symptoms. (D) identifies petechiae -- pinpoint red to purple skin discolorations that do not itch, again an incorrect identification. Correct Answer: B

A nurse is reviewing studies to answer a clinical question as part of an evidence-based practice project. The study design determines the level of evidence. Place each methodology in order from the most reliable to the least reliable. Correct 1. Meta-analysis 2. Randomized controlled trial 3. Expert opinion based on scientific principles 4. Cohort study 5. Controlled trial without randomization

Meta-analysis is a synthesis of evidence from associated randomized controlled trials. Meta-analysis is more reliable than a randomized controlled trial. Randomized controlled trials are studies where subjects randomly are assigned to a treatment or control group. A randomized control trial is more reliable than a controlled trial without randomization. Controlled trials without randomization are studies in which subjects are assigned nonrandomly to a treatment or control group. A controlled trial without randomization is more reliable than a cohort study. Cohort studies observe a group to determine the development of an outcome. Expert opinion based on principles is not based on actual evidence; it is relied on when there is no evidence from research. Topics

Central Obesity, in a prothrombotic state (prone to clots), proinflammatory state, dyslipidemia, elevated BP 135/85 These people WILL get heart disease & diabetes

Metabolic Syndrome

A client reports fatigue and dyspnea and appears pale. The nurse questions the client about medications currently being taken. In light of the symptoms, which medication causes the nurse to be most concerned? 1 Famotidine (Pepcid) 2 Methyldopa (Aldomet) 3 Ferrous sulfate (Feosol) 4 Levothyroxine (Synthroid)

Methyldopa is associated with acquired hemolytic anemia and should be discontinued to prevent progression and complications. Famotidine will not cause these symptoms; it decreases gastric acid secretion, which will decrease the risk of gastrointestinal bleeding. Ferrous sulfate is an iron supplement to correct, not cause, symptoms of anemia. Levothyroxine is not associated with red blood cell destruction.

Disease in blood vessels, in SMALLER (eyes, diabetic retinapothy)

Microangiopathy

Nursing is . . . .

NOTHING without critical thinking!

In what ways has nursing evolved?

Nursing is no longer considered a "less than" job and instead is recognized as a highly respected profession. Practice has widened to cover a wide variety of health care settings. Nurses have a specific body of knowledge. Nurses have an ethical conduct. Nurses value research and continuously publish scholarly research. Nurses don't just "care for sick people;" nurse promote health as well. Nursing is continuously growing as a highly professional discipline.

Once digested, 100% of carbohydrates are converted to glucose. However, approximately 40% of protein foods are also converted to glucose, but this has minimal effect on blood glucose levels

Once digested, what percentage of carbohydrates is converted to glucose

The P wave depicts atrial depolarization, or spread of the electrical impulse from the sinoatrial node through the atria.

P Wave

"It shows the time needed for the SA node impulse to depolarize the atria and travel through the AV node." Explanation: The PR interval is measured from the beginning of the P wave to the beginning of the QRS complex and represents the time needed for sinus node stimulation, atrial depolarization, and conduction through the AV node before ventricular depolarization. In a normal heart the impulses do not travel backward. The PR interval does not include the time it take to travel through the Purkinje fibers

P-R interval

The PR interval represents spread of the impulse through the interatrial and internodal fibers, atrioventricular node, bundle of His, and Purkinje fibers.

PR Interval

A client's infusion of normal saline infiltrated earlier today, and approximately 500 ml of saline infused into the subcutaneous tissue. The client is now complaining of excruciating arm pain and demanding "stronger pain medications." What initial action is most important for the nurse to take? A. Ask about any past history of drug abuse or addiction. B. Measure the pulse volume and capillary refill distal to the infiltration. C. Compress the infiltrated tissue to measure the degree of edema. D. Evaluate the extent of ecchymosis over the forearm area.

Pain and diminished pulse volume (B) are signs of compartment syndrome, which can progress to complete loss of the peripheral pulse in the extremity. Compartment syndrome occurs when external pressure (usually from a cast), or internal pressure (usually from subcutaneous infused fluid), exceeds capillary perfusion pressure resulting in decreased blood flow to the extremity. (A) should not be pursued until physical causes of the pain are ruled out. (C) is of less priority than determining the effects of the edema on circulation and nerve function. Further assessment of the client's ecchymosis can be delayed until the signs of edema and compression that suggest compartment syndrome have been examined (D). Correct Answer: B

Three days following surgery, a male client observes his colostomy for the first time. He becomes quite upset and tells the nurse that it is much bigger than he expected. What is the best response by the nurse? A. Reassure the client that he will become accustomed to the stoma appearance in time. B. Instruct the client that the stoma will become smaller when the initial swelling diminishes. C. Offer to contact a member of the local ostomy support group to help him with his concerns. D. Encourage the client to handle the stoma equipment to gain confidence with the procedure.

Postoperative swelling causes enlargement of the stoma. The nurse can teach the client that the stoma will become smaller when the swelling is diminished (B). This will help reduce the client's anxiety and promote acceptance of the colostomy. (A) does not provide helpful teaching or support. (C) is a useful action, and may be taken after the nurse provides pertinent teaching. The client is not yet demonstrating readiness to learn colostomy care (D). Correct Answer: B

Alternative therapy measures have become increasingly accepted within the past decade, especially in the relief of pain. Which methods qualify as alternative therapies for pain? Select all that apply. 1 Prayer 2 Hypnosis 3 Medication 4 Aromatherapy 5 Guided imagery

Prayer is an alternative therapy that may relax the client and provide strength, solace, or acceptance. The relief of pain through hypnosis is based on suggestion; also, it focuses attention away from the pain. Some clients learn to hypnotize themselves. Aromatherapy can help relax and distract the individual and thus increase tolerance for pain, as well as relieve pain. Guided imagery can help relax and distract the individual and thus increase tolerance for pain, as well as relieve pain. Analgesics, both opioid and nonopioid, long have been part of the standard medical regimen for pain relief, so they are not considered an alternative therapy.

What is the purpose of the code of ethics for nurses?

Provide a powerful statement of the ethical values, obligations, and duties of every individual who enters the nursing profession. The code of ethics serves as the nonnegotiable ethical standard of practice.

QRS always widened Treat with lidocaine Irregular rythm Can lead to vtach or vfib

Pvc

The QRS complex represents ventricular depolarization.

QRS Complex

QSEN =

Quality and safety education for nurses

Onset: 10-15 m Duration:2-4 hours Peak:1 hours Lispro (Humalog), Aspart, Apidra Give with breakfast

Rapid Acting

In developing a plan of care for a client with dementia, the nurse should remember that confusion in the elderly A. is to be expected, and progresses with age. B. often follows relocation to new surroundings. C. is a result of irreversible brain pathology. D. can be prevented with adequate sleep.

Relocation (B) often results in confusion among elderly clients--moving is stressful for anyone. (A) is a stereotypical judgment. Stress in the elderly often manifests itself as confusion, so (C) is wrong. Adequate sleep is not a prevention (D) for confusion. Correct Answer: B

Human Dignity =

Respect for inherent worth and uniqueness of individuals and populations

Nurses have a bill of rights that:

Results in advocacy on behalf of the nursing profession. Empowers nurses. Improves workplace. Ensures nurses' ability to provide safe, quality care. Allows them to freely advocate for themselves and their patients, without fear of retribution.

Regular(Humalin), Semilente Onset: 1-1.5 hr Duration: 4-6 hours Peak: 2-3 hours

Short Acting

Heart isnt beating fast enough to circulate O2, atropine

Sinus brady

The nurse is evaluating client learning about a low-sodium diet. Selection of which meal would indicate to the nurse that this client understands the dietary restrictions? A. Tossed salad, low-sodium dressing, bacon and tomato sandwich. B. New England clam chowder, no-salt crackers, fresh fruit salad. C. Skim milk, turkey salad, roll, and vanilla ice cream. D. Macaroni and cheese, diet Coke, a slice of cherry pie.

Skim milk, turkey, bread, and ice cream (C), while containing some sodium, are considered low-sodium foods. Bacon (A), canned soups (B), especially those with seafood, hard cheeses, macaroni, and most diet drinks (D) are very high in sodium. Correct Answer: C

What is a law?

Standard or rule of conduct established and enforced by government.

The nurse is performing nasotracheal suctioning. After suctioning the client's trachea for fifteen seconds, large amounts of thick yellow secretions return. What action should the nurse implement next? A. Encourage the client to cough to help loosen secretions. B. Advise the client to increase the intake of oral fluids. C. Rotate the suction catheter to obtain any remaining secretions. D. Re-oxygenate the client before attempting to suction again.

Suctioning should not be continued for longer than ten to fifteen seconds, since the client's oxygenation is compromised during this time (D). (A, B, and C) may be performed after the client is re-oxygenated and additional suctioning is performed. Correct Answer: D

Vagus stimulate, adenisone, cardiovert Narrow QRS

Svt

The T wave depicts the relative refractory period, representing ventricular repolarization

T Wave

The nurse is caring for a client who is receiving 24-hour total parenteral nutrition (TPN) via a central line at 54 ml/hr. When initially assessing the client, the nurse notes that the TPN solution has run out and the next TPN solution is not available. What immediate action should the nurse take? A. Infuse normal saline at a keep vein open rate. B. Discontinue the IV and flush the port with heparin. C. Infuse 10 percent dextrose and water at 54 ml/hr. D. Obtain a stat blood glucose level and notify the healthcare provider.

TPN is discontinued gradually to allow the client to adjust to decreased levels of glucose. Administering 10% dextrose in water at the prescribed rate (C) will keep the client from experiencing hypoglycemia until the next TPN solution is available. The client could experience a hypoglycemic reaction if the current level of glucose (A) is not maintained or if the TPN is discontinued abruptly (B). There is no reason to obtain a stat blood glucose level (D) and the healthcare provider cannot do anything about this situation. Correct Answer: C

The nurse is administering medications through a nasogastric tube (NGT) which is connected to suction. After ensuring correct tube placement, what action should the nurse take next? A. Clamp the tube for 20 minutes. B. Flush the tube with water. C. Administer the medications as prescribed. D. Crush the tablets and dissolve in sterile water.

The NGT should be flushed before, after and in between each medication administered (B). Once all medications are administered, the NGT should be clamped for 20 minutes (A). (C and D) may be implemented only after the tubing has been flushed. Correct Answer: B

Competency =

The ability to do something correctly, efficiently, and successfully.

The nurse prepares a 1,000 ml IV of 5% dextrose and water to be infused over 8 hours. The infusion set delivers 10 drops per milliliter. The nurse should regulate the IV to administer approximately how many drops per minute? A. 80 B. 8 C. 21 D. 25

The accepted formula for figuring drops per minute is: amount to be infused in one hour × drop factor/time for infusion (min)= drops per minute. Using this formula: 1,000/8 hours = 125 ml/ hour 125 × 10 (drip factor) = 1,250 drops in one hour. 1,250/ 60 (number of minutes in one hour) = 20.8 or 21 gtt/min (C). Correct Answer: C

When assisting an 82-year-old client to ambulate, it is important for the nurse to realize that the center of gravity for an elderly person is the A. Arms. B. Upper torso. C. Head. D. Feet.

The center of gravity for adults is the hips. However, as the person grows older, a stooped posture is common because of the changes from osteoporosis and normal bone degeneration, and the knees, hips, and elbows flex. This stooped posture results in the upper torso (B) becoming the center of gravity for older persons. Although (A) is a part, or an extension of the upper torso, this is not the best and most complete answer. Correct Answer: B

A male client tells the nurse that he does not know where he is or what year it is. What data should the nurse document that is most accurate? A. demonstrates loss of remote memory. B. exhibits expressive dysphasia. C. has a diminished attention span. D. is disoriented to place and time.

The client is exhibiting disorientation (D). (A) refers to memory of the distant past. The client is able to express himself without difficulty (B), and does not demonstrate a diminished attention span (C). Correct Answer: D

The nurse is teaching a client proper use of an inhaler. When should the client administer the inhaler-delivered medication to demonstrate correct use of the inhaler? A. Immediately after exhalation. B. During the inhalation. C. At the end of three inhalations. D. Immediately after inhalation.

The client should be instructed to deliver the medication during the last part of inhalation (B). After the medication is delivered, the client should remove the mouthpiece, keeping his/her lips closed and breath held for several seconds to allow for distribution of the medication. The client should not deliver the dose as stated in (A or D), and should deliver no more than two inhalations at a time (C). Correct Answer: B

An elderly male client who is unresponsive following a cerebral vascular accident (CVA) is receiving bolus enteral feedings though a gastrostomy tube. What is the best client position for administration of the bolus tube feedings? A. Prone. B. Fowler's. C. Sims'. D. Supine.

The client should be positioned in a semi-sitting (Fowler's) (B) position during feeding to decrease the occurrence of aspiration. A gastrostomy tube, known as a PEG tube, due to placement by a percutaneous endoscopic gastrostomy procedure, is inserted directly into the stomach through an incision in the abdomen for long-term administration of nutrition and hydration in the debilitated client. In (A and/or C), the client is placed on the abdomen, an unsafe position for feeding. Placing the client in (D) increases the risk of aspiration. Correct Answer: B

The nurse observes that a male client has removed the covering from an ice pack applied to his knee. What action should the nurse take first? A. Observe the appearance of the skin under the ice pack. B. Instruct the client regarding the need for the covering. C. Reapply the covering after filling with fresh ice. D. Ask the client how long the ice was applied to the skin.

The first action taken by the nurse should be to assess the skin for any possible thermal injury (A). If no injury to the skin has occurred, the nurse can take the other actions (B, C, and D) as needed. Correct Answer: A

An elderly male client who suffered a cerebral vascular accident is receiving tube feedings via a gastrostomy tube. The nurse knows that the best position for this client during administration of the feedings is A. prone. B. Fowler's. C. Sims'. D. supine.

The client should be positioned in a semi-sitting or Fowler's (B) position during feeding, in order to decrease the chance of aspiration. A gastrostomy tube, often referred to as a PEG tube, is inserted directly into the stomach through an incision in the abdomen and is used when long-term tube feedings are needed. In (A and/or C) positions, the client would be lying on his abdomen and on the tubing. In (D), the client would be lying flat on his back which would increase the chance of aspiration. Correct Answer: B

The nurse is teaching a client with numerous allergies how to avoid allergens. Which instruction should be included in this teaching plan? A. Avoid any types of sprays, powders, and perfumes. B. Wearing a mask while cleaning will not help to avoid allergens. C. Purchase any type of clothing, but be sure it is washed before wearing it. D. Pollen count is related to hay fever, not to allergens.

The client with allergies should be instructed to reduce any exposure to pollen, dust, fumes, odors, sprays, powders, and perfumes (A). The client should be encouraged to wear a mask when working around dust or pollen (B). Clients with allergies should avoid any clothing that causes itching; washing clothes will not prevent an allergic reaction to some fabrics (C). Pollen count is related to allergens (D), and the client should be instructed to stay indoors when the pollen count is high. Correct Answer: A

Which snack food is best for the nurse to provide a client with myasthenia gravis who is at risk for altered nutritional status? A. Chocolate pudding. B. Graham crackers. C. Sugar free gelatin. D. Apple slices.

The client with myasthenia gravis is at high risk for altered nutrition because of fatigue and muscle weakness resulting in dysphagia. Snacks that are semisolid, such as pudding (A) are easy to swallow and require minimal chewing effort, and provide calories and protein. (C) does not provide any nutritional value. (B and D) require energy to chew and are more difficult to swallow than pudding. Correct Answer: A

A 73-year-old female client had a hemiarthroplasty of the left hip yesterday due to a fracture resulting from a fall. In reviewing hip precautions with the client, which instruction should the nurse include in this client's teaching plan? A. In 8 weeks you will be able to bend at the waist to reach items on the floor. B. Place a pillow between your knees while lying in bed to prevent hip dislocation. C. It is safe to use a walker to get out of bed, but you need assistance when walking. D. Take pain medication 30 minutes after your physical therapy sessions.

The client's affected hip joint following a hemiarthroplasty (partial hip replacement) is at risk of dislocation for 6 months to a year following the procedure. Hip precautions to prevent dislocation include placing a pillow between the knees to maintain abduction of the hips (B). Clients should be instructed to avoid bending at the waist (A), to seek assistance for both standing and walking until they are stable on a walker or cane (C), and to take pain medication 20 to 30 minutes prior to physical therapy sessions, rather than waiting until the pain level is high after their therapy. Correct Answer: B

What is provision 1 of the code of ethics?

The nurse practices with compassion and respect for the inherent dignity, worth, and unique attributes of every person.

An elderly client who requires frequent monitoring fell and fractured a hip. Which nurse is at greatest risk for a malpractice judgment? A. A nurse who worked the 7 to 3 shift at the hospital and wrote poor nursing notes. B. The nurse assigned to care for the client who was at lunch at the time of the fall. C. The nurse who transferred the client to the chair when the fall occurred. D. The charge nurse who completed rounds 30 minutes before the fall occurred.

The four elements of malpractice are: breach of duty owed, failure to adhere to the recognized standard of care, direct causation of injury, and evidence of actual injury. The hip fracture is the actual injury and the standard of care was "frequent monitoring." (C) implies that duty was owed and the injury occurred while the nurse was in charge of the client's care. There is no evidence of negligence in (A, B, and D). Correct Answer: C

The nurse is completing a mental assessment for a client who is demonstrating slow thought processes, personality changes, and emotional lability. Which area of the brain controls these neuro-cognitive functions? A. Thalamus. B. Hypothalamus. C. Frontal lobe. D. Parietal lobe.

The frontal lobe (C) of the cerebrum controls higher mental activities, such as memory, intellect, language, emotions, and personality. (A) is an afferent relay center in the brain that directs impulses to the cerebral cortex. (B) regulates body temperature, appetite, maintains a wakeful state, and links higher centers with the autonomic nervous and endocrine systems, such as the pituitary. (D) is the location of sensory and motor functions. Correct Answer: C

During a visit to the outpatient clinic, the nurse assesses a client with severe osteoarthritis using a goniometer. Which finding should the nurse expect to measure? A. Adequate venous blood flow to the lower extremities. B. Estimated amount of body fat by an underarm skinfold. C. Degree of flexion and extension of the client's knee joint. D. Change in the circumference of the joint in centimeters.

The goniometer is a two-piece ruler that is jointed in the middle with a protractor-type measuring device that is placed over a joint as the individual extends or flexes the joint to measure the degrees of flexion and extension on the protractor (C). A doppler is used to measure blood flow (A). Calipers are used to measure body fat (B). A tape measure is used to measure circumference of body parts (D). Correct Answer: C

A female client with a nasogastric tube attached to low suction states that she is nauseated. The nurse assesses that there has been no drainage through the nasogastric tube in the last two hours. What action should the nurse take first? A. Irrigate the nasogastric tube with sterile normal saline. B. Reposition the client on her side. C. Advance the nasogastric tube an additional five centimeters. D. Administer an intravenous antiemetic prescribed for PRN use.

The immediate priority is to determine if the tube is functioning correctly, which would then relieve the client's nausea. The least invasive intervention, (B), should be attempted first, followed by (A and C), unless either of these interventions is contraindicated. If these measures are unsuccessful, the client may require an antiemetic (D). Correct Answer: B

A client is to receive cimetidine (Tagamet) 300 mg q6h IVPB. The preparation arrives from the pharmacy diluted in 50 ml of 0.9% NaCl. The nurse plans to administer the IVPB dose over 20 minutes. For how many ml/hr should the infusion pump be set to deliver the secondary infusion?

The infusion rate is calculated as a ratio proportion problem, i.e., 50 ml/ 20 min : x ml/ 60 min. Multiply extremes and means 50 × 60 /20x 1= 300/20=150 Correct Answer: 150

An unlicensed assistive personnel (UAP) places a client in a left lateral position prior to administering a soap suds enema. Which instruction should the nurse provide the UAP? A. Position the client on the right side of the bed in reverse Trendelenburg. B. Fill the enema container with 1000 ml of warm water and 5 ml of castile soap. C. Reposition in a Sim's position with the client's weight on the anterior ilium. D. Raise the side rails on both sides of the bed and elevate the bed to waist level.

The left sided Sims' position allows the enema solution to follow the anatomical course of the intestines and allows the best overall results, so the UAP should reposition the client in the Sims' position, which distributes the client's weight to the anterior ilium (C). (A) is inaccurate. (B and D) should be implemented once the client is positioned. Correct Answer: C

A client who is in hospice care complains of increasing amounts of pain. The healthcare provider prescribes an analgesic every four hours as needed. Which action should the nurse implement? A. Give an around-the-clock schedule for administration of analgesics. B. Administer analgesic medication as needed when the pain is severe. C. Provide medication to keep the client sedated and unaware of stimuli. D. Offer a medication-free period so that the client can do daily activities.

The most effective management of pain is achieved using an around-the-clock schedule that provides analgesic medications on a regular basis (A) and in a timely manner. Analgesics are less effective if pain persists until it is severe, so an analgesic medication should be administered before the client's pain peaks (B). Providing comfort is a priority for the client who is dying, but sedation that impairs the client's ability to interact and experience the time before life ends should be minimized (C). Offering a medication-free period allows the serum drug level to fall, which is not an effective method to manage chronic pain (D). Correct Answer: A

The nurse observes an unlicensed assistive personnel (UAP) taking a client's blood pressure with a cuff that is too small, but the blood pressure reading obtained is within the client's usual range. What action is most important for the nurse to implement? A. Tell the UAP to use a larger cuff at the next scheduled assessment. B. Reassess the client's blood pressure using a larger cuff. C. Have the unit educator review this procedure with the UAPs. D. Teach the UAP the correct technique for assessing blood pressure.

The most important action is to ensure that an accurate BP reading is obtained. The nurse should reassess the BP with the correct size cuff (B). Reassessment should not be postponed (A). Though (C and D) are likely indicated, these actions do not have the priority of (B). Correct Answer: B

An obese male client discusses with the nurse his plans to begin a long-term weight loss regimen. In addition to dietary changes, he plans to begin an intensive aerobic exercise program 3 to 4 times a week and to take stress management classes. After praising the client for his decision, which instruction is most important for the nurse to provide? A. Be sure to have a complete physical examination before beginning your planned exercise program. B. Be careful that the exercise program doesn't simply add to your stress level, making you want to eat more. C. Increased exercise helps to reduce stress, so you may not need to spend money on a stress management class. D. Make sure to monitor your weight loss regularly to provide a sense of accomplishment and motivation.

The most important teaching is (A), so that the client will not begin a dangerous level of exercise when he is not sufficiently fit. This might result in chest pain, a heart attack, or stroke. (B, C, and D) are important instructions, but are of less priority than (A). Correct Answer: A

What is provision 8 of the code of ethics?

The nurse collaborate with other health professionals and the public to protect human rights, promote health diplomacy, and reduce health disparities.

An elderly resident of a long-term care facility is no longer able to perform self-care and is becoming progressively weaker. The resident previously requested that no resuscitative efforts be performed, and the family requests hospice care. What action should the nurse implement first? A. Reaffirm the client's desire for no resuscitative efforts. B. Transfer the client to a hospice inpatient facility. C. Prepare the family for the client's impending death. D. Notify the healthcare provider of the family's request.

The nurse should first communicate with the healthcare provider (D). Hospice care is provided for clients with a limited life expectancy, which must be identified by the healthcare provider. (A) is not necessary at this time. Once the healthcare provider supports the transfer to hospice care, the nurse can collaborate with the hospice staff and healthcare provider to determine when (B and C) should be implemented. Correct Answer: D

The nurse witnesses the signature of a client who has signed an informed consent. Which statement best explains this nursing responsibility? A. The client voluntarily signed the form. B. The client fully understands the procedure. C. The client agrees with the procedure to be done. D. The client authorizes continued treatment.

The nurse signs the consent form to witness that the client voluntarily signs the consent (A), that the client's signature is authentic, and that the client is otherwise competent to give consent. It is the healthcare provider's responsibility to ensure the client fully understands the procedure (B). The nurse's signature does not indicate (C or D). Correct Answer: A

What is provision 2 of the code of ethics?

The nurse's primary commitment is to the patient, whether an individual, family, group, community, or population.

What is provision 7 of the code of ethics?

The nurse, in all roles and settings, advance the profession through research and scholarly inquiry, professional standards development, and the generation of both nursing and health policy.

What is provision 6 of the code of ethics?

The nurse, through individual and collective effort, establishes, maintains, and improves the ethical environment of the work setting and conditions of employment that are conducive to safe, quality health care.

What are some of the ANA standards of nursing practice?

The nursing process Ethics Education EBP/Research Quality of Practice Communication Leadership Collaboration Professional Practice Eval Resource Utilization Environmental Health Collegiality

Regardless of the various definitions of nursing, what is the central focus of *ALL* definitions?

The patient.

When assessing a client with wrist restraints, the nurse observes that the fingers on the right hand are blue. What action should the nurse implement first? A. Loosen the right wrist restraint. B. Apply a pulse oximeter to the right hand. C. Compare hand color bilaterally. D. Palpate the right radial pulse.

The priority nursing action is to restore circulation by loosening the restraint (A), because blue fingers (cyanosis) indicates decreased circulation. (C and D) are also important nursing interventions, but do not have the priority of (A). Pulse oximetry (B) measures the saturation of hemoglobin with oxygen and is not indicated in situations where the cyanosis is related to mechanical compression (the restraints). Correct Answer: A

What is provision 9 of the code of ethics?

The profession of nursing, collectively through its professional organizations, must articulate nursing values, maintain the integrity of the profession, and integrate principles of social justice into nursing and health policy.

An adult male client with a history of hypertension tells the nurse that he is tired of taking antihypertensive medications and is going to try spiritual meditation instead. What should be the nurse's first response? A. It is important that you continue your medication while learning to meditate. B. Spiritual meditation requires a time commitment of 15 to 20 minutes daily. C. Obtain your healthcare provider's permission before starting meditation. D. Complementary therapy and western medicine can be effective for you.

The prolonged practice of meditation may lead to a reduced need for antihypertensive medications. However, the medications must be continued (A) while the physiologic response to meditation is monitored. (B) is not as important as continuing the medication. The healthcare provider should be informed, but permission is not required to meditate (C). Although it is true that this complimentary therapy might be effective (D), it is essential that the client continue with antihypertensive medications until the effect of meditation can be measured. Correct Answer: A

Who defines the legal scope of practice for nursing?

The state board of a specific state.

What's the ANA's definition of nursing?

The protection, promotion, and optimization of health and abilities, prevention of illness and injury, alleviation of suffering through the diagnosis and treatment of human response, and advocacy in the care of individuals, families, communities, and populations.

Utilitarian =

The rightness or wrongness of an action depends on the consequences of the action.

The nurse assesses a client's pulse and documents the strength of the pulse as 3+. The nurse understands that this indicates the pulse is: 1 faint, barely detectable. 2 slightly weak, palpable. 3 normal. 4 bounding.

The strength of a pulse is a measurement of the force at which blood is ejected against the arterial wall. Palpation should be done using the fingertips and intensity of the pulse graded on a scale of 0 to 4 + with 0 indicating no palpable pulse, 1 + indicating a faint, but detectable pulse, 2 + suggesting a slightly more diminished pulse than normal, 3 + is a normal pulse, and 4 + indicating a bounding pulse.

While instructing a male client's wife in the performance of passive range-of-motion exercises to his contracted shoulder, the nurse observes that she is holding his arm above and below the elbow. What nursing action should the nurse implement? A. Acknowledge that she is supporting the arm correctly. B. Encourage her to keep the joint covered to maintain warmth. C. Reinforce the need to grip directly under the joint for better support. D. Instruct her to grip directly over the joint for better motion.

The wife is performing the passive ROM correctly, therefore the nurse should acknowledge this fact (A). The joint that is being exercised should be uncovered (B) while the rest of the body should remain covered for warmth and privacy. (C and D) do not provide adequate support to the joint while still allowing for joint movement. Correct Answer: A

A male client being discharged with a prescription for the bronchodilator theophylline tells the nurse that he understands he is to take three doses of the medication each day. Since, at the time of discharge, timed-release capsules are not available, which dosing schedule should the nurse advise the client to follow? A. 9 a.m., 1 p.m., and 5 p.m. B. 8 a.m., 4 p.m., and midnight. C. Before breakfast, before lunch and before dinner. D. With breakfast, with lunch, and with dinner.

Theophylline should be administered on a regular around-the-clock schedule (B) to provide the best bronchodilating effect and reduce the potential for adverse effects. (A, C, and D) do not provide around-the-clock dosing. Food may alter absorption of the medication (D). Correct Answer: B

An elderly client with a fractured left hip is on strict bedrest. Which nursing measure is essential to the client's nursing care? A. Massage any reddened areas for at least five minutes. B. Encourage active range of motion exercises on extremities. C. Position the client laterally, prone, and dorsally in sequence. D. Gently lift the client when moving into a desired position.

To avoid shearing forces when repositioning, the client should be lifted gently across a surface (D). Reddened areas should not be massaged (A) since this may increase the damage to already traumatized skin. To control pain and muscle spasms, active range of motion (B) may be limited on the affected leg. The position described in (C) is contraindicated for a client with a fractured left hip. Correct Answer: D

What is the purpose of HIPAA?

To ensure patient rights.

A client is in the radiology department at 0900 when the prescription levofloxacin (Levaquin) 500 mg IV q24h is scheduled to be administered. The client returns to the unit at 1300. What is the best intervention for the nurse to implement? A. Contact the healthcare provider and complete a medication variance form. B. Administer the Levaquin at 1300 and resume the 0900 schedule in the morning. C. Notify the charge nurse and complete an incident report to explain the missed dose. D. Give the missed dose at 1300 and change the schedule to administer daily at 1300.

To ensure that a therapeutic level of medication is maintained, the nurse should administer the missed dose as soon as possible, and revise the administration schedule accordingly to prevent dangerously increasing the level of the medication in the bloodstream (D). The nurse should document the reason for the late dose, but (A and C) are not warranted. (B) could result in increased blood levels of the drug. Correct Answer: D

What are some patient rights?

To see and copy their health record. To update their health record. To request correction of any mistakes. To get a list of disclosures. To request restrictions on certain uses or disclosures. To choose how to receive health information

What's an ethical dilemma? Professor Donadio's example of the PT on the vent.

Two (or more) clear moral principles apply, but support mutually inconsistent courses of actions.

Treat: Sulfonurea (Increase insulin) + biguanide (incr. isnulin sensitivity), diet & exercise

Type 2

TX: insulin, meal spacing

Type I

The healthcare provider prescribes morphine sulfate 4mg IM STAT. Morphine comes in 8 mg per ml. How many ml should the nurse administer? A. 0.5 ml. B. 1 ml. C. 1.5 ml. D. 2 ml.

Using ratio and proportion: 8mg: 1ml :: 4mg:Xml 8X=4 X=0.5 Correct Answer: A

What are the two basic theoretical frameworks of ethics?

Utilitarian and Deontologic

What is the most important reason for starting intravenous infusions in the upper extremities rather than the lower extremities of adults? A. It is more difficult to find a superficial vein in the feet and ankles. B. A decreased flow rate could result in the formation of a thrombosis. C. A cannulated extremity is more difficult to move when the leg or foot is used. D. Veins are located deep in the feet and ankles, resulting in a more painful procedure.

Venous return is usually better in the upper extremities. Cannulation of the veins in the lower extremities increases the risk of thrombus formation (B) which, if dislodged, could be life-threatening. Superficial veins are often very easy (A) to find in the feet and legs. Handling a leg or foot with an IV (C) is probably not any more difficult than handling an arm or hand. Even if the nurse did believe moving a cannulated leg was more difficult, this is not the most important reason for using the upper extremities. Pain (D) is not a consideration. Correct Answer: B

Rate is verrrry fast Defib, acls, amiodarone

Vfib

A client who is 5' 5" tall and weighs 200 pounds is scheduled for surgery the next day. What question is most important for the nurse to include during the preoperative assessment? A. What is your daily calorie consumption? B. What vitamin and mineral supplements do you take? C. Do you feel that you are overweight? D. Will a clear liquid diet be okay after surgery?

Vitamin and mineral supplements (B) may impact medications used during the operative period. (A and C) are appropriate questions for long-term dietary counseling. The nature of the surgery and anesthesia will determine the need for a clear liquid diet (D), rather than the client's preference. Correct Answer: B

Hr is 150-250 no p wave (cant determine atrial rate) No Pulse: defibrillator, amiodarone, cpr, acls Pulse: cardiovert/amioderano

Vtach

What is moral distress?

When the nurse knows the right thing to do but either personal or institutional factors make it difficult to do the right thing.

0.24 seconds Explanation: In adults, the normal range for the PR is 0.12 to 0.20 seconds. A PR internal of 0.24 seconds would indicate a first-degree heart block.

Which PR interval presents a first-degree heart block?

Immediate bystander CPR Explanation: The treatment of choice for v-fib is immediate bystander cardiopulmonary resuscitation (CPR), defibrillation as soon as possible, and activation of emergency services

Which of the following is the treatment of choice for ventricular fibrillation

What's a crime?

Wrong against person/property & the public

Can an RN's license be suspended/revoked?

Yes! If said RN becomes carless, reckless, and negligent of ones career and patients.

Immediate defibrillation Explanation: Defibrillation is used during pulseless ventricular tachycardia, ventricular fibrillation, and asystole (cardiac arrest) when no identifiable R wave is present.

You enter your client's room and find him pulseless and unresponsive. What would be the treatment of choice for this client?

used to treat symptomatic bradycardia

atropine

Beneficence =

benefit the patient

Since Florence Nightingale, nursing has . . .

broadened in all areas.

Altruism =

concern for welfare and well-being of others

Early morning hyperglycemia resulting from increased growth hormone circulation

dawn phenomenon

Reflective practice is . . .

essential to professional practice.

Bioethics =

ethics dealing with human lives

The healthcare provider prescribes 1,000 ml of Ringer's Lactate with 30 Units of Pitocin to run in over 4 hours for a client who has just delivered a 10 pound infant by cesarean section. The tubing has been changed to a 20 gtt/ml administration set. The nurse plans to set the flow rate at how many gtt/min? A. 42 gtt/min. B. 83 gtt/min. C. 125 gtt/min. D. 250 gtt/min.

gtt/min = 20gtts/ml X 1000 ml/4hrs X 1 hr/60 min Correct Answer: B

Disparities =

inequality

Torts may be . . .

intentional or unintentional.

Nursing is also,

patent-centered, meaning the patient drives the entire process.

Impaired glucose tolerance 140-199 impaired fasting glucose 110-126 screen at 40 is FHx present encourage weightloss

pre-diabetes


Set pelajaran terkait

Oklahoma Life and Health Exam Study Guide Part 3

View Set

NCLEX Child Health- Renal and Urinary

View Set

Religion Chapter 6-Abraham is Our Father in Faith

View Set

Cell Biology Chapter 12: Membrane Transport I

View Set

Kenneth Burke: Semantic and Poetic Meaning

View Set

Operant Conditioning and Behavior Management

View Set

Skeletal: Main Cranial Bones and Sutures, Fontanels...

View Set